Jump to content

Wikipedia:Reference desk/Humanities

From Wikipedia, the free encyclopedia

This is an old revision of this page, as edited by Chaela89 (talk | contribs) at 19:01, 9 September 2008. The present address (URL) is a permanent link to this revision, which may differ significantly from the current revision.

Welcome to the humanities section
of the Wikipedia reference desk.
Select a section:
Want a faster answer?

Main page: Help searching Wikipedia

   

How can I get my question answered?

  • Select the section of the desk that best fits the general topic of your question (see the navigation column to the right).
  • Post your question to only one section, providing a short header that gives the topic of your question.
  • Type '~~~~' (that is, four tilde characters) at the end – this signs and dates your contribution so we know who wrote what and when.
  • Don't post personal contact information – it will be removed. Any answers will be provided here.
  • Please be as specific as possible, and include all relevant context – the usefulness of answers may depend on the context.
  • Note:
    • We don't answer (and may remove) questions that require medical diagnosis or legal advice.
    • We don't answer requests for opinions, predictions or debate.
    • We don't do your homework for you, though we'll help you past the stuck point.
    • We don't conduct original research or provide a free source of ideas, but we'll help you find information you need.



How do I answer a question?

Main page: Wikipedia:Reference desk/Guidelines

  • The best answers address the question directly, and back up facts with wikilinks and links to sources. Do not edit others' comments and do not give any medical or legal advice.
See also:


September 3

VPs in the USA

Are there any historical precedents in the US where a VP has been picked by the presidential candidate of a major party, and then dropped and another person chosen to stand instead? Thanks for any info, Alex --AlexSuricata (talk) 00:05, 3 September 2008 (UTC)[reply]

Thomas Eagleton. -- JackofOz (talk) 00:12, 3 September 2008 (UTC)[reply]

granny smiths

(troll question removed). Edison2 (talk) 01:33, 3 September 2008 (UTC)[reply]

Czechoslovak Communist Exiles

Recently, I have been trying to research Communist history, namely in Czechoslovakia. Is there a Wikipedia article about the people who went abroad and were stripped of their citizenship while they weren't in Czechoslovakia? Also, are there any good online links? I'd like to know how they managed to get their citizenship back, get a passport from another country, etc. Vltava 68 (talk, contribs) 10:19, 3 September 2008 (UTC)[reply]

I'm not sure if there is a Wikipedia article on the subject, but if you're up for some research, you might find some candidates in Category:Czech expatriates and Category:Slovak expatriates. — Twas Now ( talkcontribse-mail ) 10:29, 3 September 2008 (UTC)[reply]

soranus

i am looking for a high quality image of the bust of soranus, greek gyneacologist. or indeed for the original or a copy of it? i have a client wishing a charcoal drawing of this and my research so far has only thrown up small poor quality copies. thank you. 89.159.144.169 (talk) 12:40, 3 September 2008 (UTC)[reply]

There is a photo of a bust of Soranus at [1], but I do not know if it is of sufficiently high quality for your purposes. DuncanHill (talk) 12:57, 3 September 2008 (UTC)[reply]

Etiquette question

I know a woman who will be giving birth in a few months, at which time I look to send a card of congratulations. I've got a problem, though: I don't know her husband's name, so I'm not sure how I'll address the letter. She's Mrs X-Y, although she normally goes by Mrs X only. Is he likely to be Mr X, or Mr Y, or is it really impossible to know without asking one of them? 63.172.28.202 (talk) 13:47, 3 September 2008 (UTC)[reply]

I'm rather lucky. I work with people from all over the world. So, I can cheat and ask, "How do you spell your last name?" Even if it is something like "Wong", I can reply, "Oh, you spell it the normal way. That's easy to remember." -- kainaw 13:51, 3 September 2008 (UTC)[reply]
(edit conflict) If she's Mrs. X-Y, I would guess that he's probably Mr. Y, but there's no way to say for sure. Could you ask somebody who's closer to Mrs. X-Y, and might know the answer? (BTW, my wife did not change her name when we married, and we often get cards with incorrect names. I'm sure that they will appreciate the kind thoughts even if you make a mistake.) -- Coneslayer (talk) 13:53, 3 September 2008 (UTC)[reply]
Another thought--There may be a birth announcement in the local paper, with their full names. -- Coneslayer (talk) 13:55, 3 September 2008 (UTC)[reply]
Agree with what Coneslayer said, but also, will they not send you an announcement card with their names on? DuncanHill (talk) 13:59, 3 September 2008 (UTC)[reply]
At least here in the U.S., such announcements (of graduations, marriages, births) are often seen as solicitations for gifts. Therefore, polite people may choose to send them only to their closest family and friends, to avoid the appearance of "gift-grubbing". Mr./Ms. 63.172.28.202 might be outside this close circle. -- Coneslayer (talk) 14:09, 3 September 2008 (UTC)[reply]
I think you'll have to ask someone, there are so many possibilities. She could use a double barrelled name because she's combined her maiden name and her husband's name, or her parents may have combined their names (or some earlier generation), or her husband's parents (or higher generation) could have combined their names, and she's taken her husband's name. (There may even be other possibilities, but I think those are the most likely.) You could just avoid surnames entirely, either by using first names (if you are familiar enough with them), or no names at all, "To the proud parents of XYZ, congratulations and best wishs, [your name here]". It's difficult to know how offended they will be if you use the wrong names. I doubt many people would be offended at being asked, though, just be appropriately apologetic about it (or ask someone else that's likely to know). --Tango (talk) 18:24, 3 September 2008 (UTC)[reply]
Assuming you know their first names, just put those on the card. DJ Clayworth (talk) 22:24, 3 September 2008 (UTC)[reply]
I'll probably have to end up asking: as far as I know, I don't know anyone but her who knows him, and I'm planning on sending this card in the mail, so the postal worker likely will want more than their first names :-) 63.172.28.202 (talk) 05:07, 4 September 2008 (UTC)[reply]
Why would the postal worker care who it's addressed to? They just go by the address. --Tango (talk) 05:11, 4 September 2008 (UTC)[reply]

Try this: send a "welcome to the world" card to the child, instead. DOR (HK) (talk) 09:21, 5 September 2008 (UTC)[reply]

The child is probably unnamed at this point, though, so then the original poster might again need to know which surname to use. Personally, I'd just ask. It's going to be a hell of a lot less embarrassing than getting the name wrong, and if you don't know him that well, it's by no means impolite to ask. There's no reason to expect that you would offend them by asking. -- Captain Disdain (talk) 22:25, 6 September 2008 (UTC)[reply]

Different Amounts for Different Charges?

In the US, or if you want to be specific, Connecticut, how much illegal substances (marijuana) do you have to have on your person to be convicted of a minor charge? How much for a major charge? Same question for controlled substances like oxycodone? --Anilmanohar (talk) 15:04, 3 September 2008 (UTC)[reply]

Try here. (first Google hit for 'connecticut marijuana possession') Prince of Canada t | c 17:17, 3 September 2008 (UTC)[reply]
Is this perhaps legal advice? 63.172.28.202 (talk) 05:10, 4 September 2008 (UTC)[reply]
Legal advice is like, "how should I plead in court", not "what exactly does the law say on this specific issue." The former needs expert advice. The latter is just about looking up a statute. --98.217.8.46 (talk) 12:32, 4 September 2008 (UTC)[reply]

Amazonian tribe

Which Amazonian tribe does this practise where it is like a celebration where the men hold a turtle and have to hold it from the ladies and the ladies pinch them, bite them or whatever they do to the men in order to get the turtle? This is in Brazil and I saw this thing on t.v. —Preceding unsigned comment added by 142.204.74.121 (talk) 16:03, 3 September 2008 (UTC)[reply]

Preclassical statuary depictions of masturbation

I and some others am trying to bring the Masturbation article to GA status. Among the uncited statements is this one:


I'm no expert in archaeology, and I'm having trouble finding reliable sources for the former statement. If this figurine exists, it should be referenced somewhere. Can anyone throw me a clue?

(Good suggestions for the second sentence are also welcome, though these are easier-than-impossible to find so far.) The Wednesday Island (talk) 17:45, 3 September 2008 (UTC)[reply]

Apparently it was found at Hagar Qim. Try googling that plus masturbation. Adam Bishop (talk) 23:43, 3 September 2008 (UTC)[reply]
This article from the Council for British Archaeology may be of interest (scroll down a bit to find the right section) [2] DuncanHill (talk) 01:02, 4 September 2008 (UTC)[reply]
Wonderful-- just what was needed. Thank you. The Wednesday Island (talk) 11:58, 4 September 2008 (UTC)[reply]

Too much of a good thing?

My friend and I were talking about credit cards and he said that paying your credit card statement on time, every time, can actually be a bad thing because you won't make credit card companies any money, therefore it would essentially lower one's credit score. I sort of see the sense in it, but I don't really believe it, because they make money through the merchants, right?... [Disclaimer: I'm not seeking legal/financial advice] -- MacAddct  1984 (talk • contribs) 17:52, 3 September 2008 (UTC)[reply]

I certainly have a very hard time believing paying your credit card statement on time, every time, could have any sort of harmful effect whatsoever. JIP | Talk 17:54, 3 September 2008 (UTC)[reply]
Your credit score goes down if you are late (by even a single day) when paying your bill. All you need to do is ask for your credit report. There is a section that lists every time you've been late paying a bill (and the creditor reports it). There is no section listing how often you pay on time. -- kainaw 17:59, 3 September 2008 (UTC)[reply]
Pay your bill on time if possible. Not only is letting balances build up one of the worst financial mistakes to make, but paying on time (and in full) enhances your credit score. As long as the credit card / phone bill /etc is in your name you are building your credit worthiness. That is what institutions will look at when deciding to lend you money (or the rate at which they will!) in the future. Also, Credit Rating Agencies are separate from credit card companies. And you are correct, credit cards charge merchants a percentage fee per transaction and also on the interest rates charged customers on outstanding balances. —Preceding unsigned comment added by Bwebster99 (talkcontribs) 18:16, 3 September 2008 (UTC)[reply]
You should certainly pay at least the minimum on time every time, otherwise you will do serious harm to your credit score. I guess the question you're asking is if you should pay the balance in full every time. I think you should, the cost of the interest (which is usually very high on credit cards - if it's a 0% card, it's another matter entirely) far outweighs the fact that the credit card company won't like you much (they still make money though, the shops you use your credit card in do give the credit card company a percentage). Credit card companies certainly aren't fans of people that are good with money, but I think it's rare for them to actually do anything to penalise you. --Tango (talk) 18:29, 3 September 2008 (UTC)[reply]
The minimum you should do is pay the minimum of your credit card payment on time, every time. Even the slightest deviation will harm your credit card statement. I have had a credit card for almost five years and have managed to do this every time. The more you pay of your credit card payment, the better. The credit card companies certainly want you to use your credit card very much - but they also except you to pay up every time. Failing to pay up is far worse than spending too little. And also, beware of these new-fangled "instant loan" companies that offer to get rid of credit card debt fast. They often have interest rates much higher than the credit card companies do. JIP | Talk 19:24, 3 September 2008 (UTC)[reply]
You've gotten a lot of good advice regarding your credit score, but your friend may have been thinking of something else... a profit score. (Sacrificing your credit score to help your profit score is probably a lousy idea, BTW.) -- Coneslayer (talk) 19:33, 3 September 2008 (UTC)[reply]
Here's one possibility: it is possible in my experience, at least in the US, to have no credit score at all if you've never been lent money. This can be problematic if one day you want to use your credit score for something. If you pay off your credit card every month, perhaps this counts as never having being lent money? Marnanel (talk) 19:47, 3 September 2008 (UTC)[reply]
Having no, or a short, credit history can indeed be a problem. I think having a credit card would help in that respect as long as you use it, regardless of whether you ever actually borrow on it. You are being lent money for the month or so it takes you to pay it back and paying it back in full every time shows you are able to be responsible with your spending, which will help your credit score. Having the card and never buying anything on it may not count as having a long credit history (it could also hurt your score because you have more available credit - banks don't like to lend money to people that already have lots of available credit since they could easily end up with more debt than they can pay off, even if they don't have any debt now). --Tango (talk) 20:51, 3 September 2008 (UTC)[reply]
Speaking of too much of a good thing, the Federal Reserve consumer credit estimate for June 2008 says that American consumers have $986 billion in outstanding revolving credit, and $1.618 trillion in non-revolving credit (e.g., auto loans, education loans, boat loans). That's $2.586 trillion. — OtherDave (talk) 21:39, 3 September 2008 (UTC)[reply]
Which is nearly $8500 per person (which includes children who hopefully have no debt, and plenty of people will have mortgages on top of that). --Tango (talk) 22:43, 3 September 2008 (UTC)[reply]
Thanks for all the replies, and thank you for that link Coneslayer, that seems to cover what I was talking about. I guess I fall into the "not that profitable for lenders, but got potential"-category. I just never understood the concept of spending money that you don't have... -- MacAddct  1984 (talk • contribs) 22:50, 3 September 2008 (UTC)[reply]
Spending money you don't have can be a good decision in some cases (buying a house is tricky otherwise, as is going through university, in other cases it may be a good decision now, but only because you made bad decisions earlier). Borrowing on a credit card is never a good decision - if you're that strapped for cash, find a good unsecured personal loan, you can get much better interest rates. (Of course, if you can get an interest free credit card, and trust yourself not to forget (or be unable) to pay it off at the end of the year, or however long it's free for, then go for it!) --Tango (talk) 23:00, 3 September 2008 (UTC)[reply]
Borrowing on a credit card can be a very good thing if you are looking for insurance on the things you purchase and the transaction that takes place, especially if you are not 100% sure of the person you are paying. Using a credit card on the internet means that it is the creditors' money that is stolen if something goes wrong and not yours. Also, if you pay your credit card immediately, you don't build up any interest. Never say never. ;) - Phydaux (talk) 15:00, 4 September 2008 (UTC)[reply]
Sorry, that's my imprecise use of language at fault - by "borrowing", I mean for longer than one statement period (shorter than that is still technically borrowing, but that's not your reason for using a credit card). Purchasing things on a credit card has significant advantages, but you should always pay it off in full before they start charging interest. --Tango (talk) 21:50, 4 September 2008 (UTC)[reply]

As an American living abroad all of my adult life, I was unhappy (but not surprised) to find out I had no credit rating at all. It was suggested that I take out a credit card and make only the minimum payment for three months, and then pay the full debt. That would establish top-quality credit. Of course, carrying any credit card debt at all is incredibly stupid for most people, as the interest rate is at least 3 and sometimes 10 times what you make keeping the money on deposit. DOR (HK) (talk) 09:26, 5 September 2008 (UTC)[reply]


September 4

Hockey Moms

Question for US people: Sarah Palin is just speaking on my TV at the moment. She seems to be going on a bit about "Hockey Moms" and there's a joke too and proud Hockey Moms in the audience with banners, etc. I'm not American: are these "Hockey Moms" just simply women whose children play hockey, or is there some deeper meaning/identity there? (there's no article on them) Thanks for info, Alex --AlexSuricata (talk) 02:50, 4 September 2008 (UTC)[reply]

I'm not American but it sounds like a variation of Soccer mom. That term is famous in US and I guess Americans would associate to that. PrimeHunter (talk) 02:59, 4 September 2008 (UTC)[reply]
A Soccer Mom is a similar creature, but soccer fields tend to be more accessible than ice rinks, because they are cheaper to maintain, for a start. And Soccer Moms have occasions when they are likely warm. A Hockey Mom is always cold. A Hockey Mom is not just one whose children play ice hockey, but one who drives them to practices and games, watches them play, talks hockey with other moms, is part of a car pool that takes several children to "away" games . . . What you may not know is that much of ice hockey happens before school, so it isn't unusual for a Hockey Mom to be picking up kids at 6:00 a.m. and then taking them from the rink home for showers and then to school. It is a big commitment of time and there is a lot of driving involved, especially if you live outside a city and thus have little or no public transportation. Hockey equipment takes up a lot of space and tends to be heavy (helmets, pads, ice skates, sticks) so Hockey Moms are also porters, as well as laundry managers. And these kids start really young, often when they are in kindergarten (ages 4 and 5) so there are a lot of years involved, too. There are also Hockey Dads especially as the children (usually boys) get older. This is terminology more recognizable in Canada than in the lower 48, but hockey is big in Alaska. ៛ Bielle (talk) 03:10, 4 September 2008 (UTC)[reply]
Sounds like dedicated mothers who are also angry, distraught and less than rational because they are sleep-deprived. Edison (talk) 04:42, 4 September 2008 (UTC)[reply]
Possibly worth mentioning that "hockey" means ice hockey in the US, not field hockey. The Wednesday Island (talk) 11:57, 4 September 2008 (UTC)[reply]
I'd agree that for the most part it's an extension of soccer mom. Both these terms tend to imply all-American, non-elite, family-first, unpretentious people. (There's a whiff of sexism, in that "soccer dad" is rarer by far.) I have the totally unfounded impression that "hockey mom" implies more blue in the collar, in the U.S., than "soccer mom" does. Far fewer Americans (children or adults) play hockey, as a percentage of the population, than Canadians do. And see, very tangentially, Michael Melski's play Hockey Mom, Hockey Dad, set in northern Nova Scotia. — OtherDave (talk) 15:02, 4 September 2008 (UTC)[reply]
Among other attributes mentioned, I tend to think of Soccer Moms as being susceptible to, and/or users of, "think of the children" arguments. -- Coneslayer (talk) 17:58, 4 September 2008 (UTC)[reply]
My guess is that it is the Alaskan equivalent of soccer mom. Soccer mom is the more recognizable term, but changing the sport to Hockey would be understandable to most people in context. --98.217.8.46 (talk) 12:21, 4 September 2008 (UTC)[reply]
Soccer moms, and therefore by extension hockey moms, would also tend to drive minivans which can hold a large number of kids plus all of their equipment. Corvus cornixtalk 18:32, 4 September 2008 (UTC)[reply]
So the overarching implication is that Hockey Moms, and Soccer Moms are parents who are dedicated to their children (and by extension, families) and who are willing to sacrifice their own time, money and leisure for their children's sport, development, etc. It further implies that a Hockey Mom will be concerned with issues that relate to children, families and suburbia, and therefore probably (although not implicitly) a collection of usually conservative views with regards to family protection, abortion, sexuality and religion. Steewi (talk) 04:35, 8 September 2008 (UTC)[reply]

Having grown up in Alaska, and living now as I do in the Upper Peninsula of Michigan, I have been well-acquainted with Hockey Moms much of my life. As has been mentioned, Hockey Moms are similar to Soccer Moms, in that they drive minivans loaded with kids and equipment all over the place, and are the classic stereotype of a Middle Class housewife. However, there is a degree of difference between Hockey and Soccer Moms, in that Hockey Moms are more noted for their aggressive behavior at games. This difference, I believe, is what Palin refered to in comparing Hockey Moms to bulldogs. Saukkomies 15:30, 11 September 2008 (UTC)[reply]

Perceived taste relating to expectation

What is the name in psychology for the thinking-something-tastes-better-because-it-should-taste-better effect?:

Lanma726 (talk) 03:35, 4 September 2008 (UTC)[reply]

I'd suggset looking at something like List of biases and trying to find the best match. I suspect that people would consider these a bias as it seems it would be caused by 'information' influencing the way something is perceived (i.e. marketing, peer pressure, etc.) 194.221.133.226 (talk) 15:42, 4 September 2008 (UTC)[reply]
That sounds psychosomatic to me... or it would if it were an illness. - Lambajan 02:49, 5 September 2008 (UTC)[reply]

Weird hearing

Ok, explain this: My younger brother used to have a toy telephone that, when you pressed different buttons, made different sounds. One of the buttons had a picture of an ambulance, which, when pressed, made a voice say "Emergency! Emergency!", which makes perfect sense. My Dad, however, was terribly confused by this button because he was convinced it was saying "Liverpool! Liverpool". After being told what it really said he was, eventually, able to hear it correctly. Similarly, after being told what he heard, other people (myself included) were able to make it sound like that too (I'd liken it to making a Multistable perception image switch between the two possibilities at will). Is this a known phenomenon? --Tango (talk) 04:41, 4 September 2008 (UTC)[reply]

A mondegreen is a known phenomenon, although that is a recent coinage. I'm not sure how much mishearings were appreciated before that.--Shantavira|feed me 07:48, 4 September 2008 (UTC)[reply]
I was thinking of mondegreens too, but that really only applies to a song lyric, line from a poem, or book or movie title etc. In a general context, I'd say it's just a mishearing. People with hearing issues, and their families and friends, would be well aware of this phenomenon, although the auditory interpretation of "Emergency" as "Liverpool" seems an extreme case. -- JackofOz (talk) 09:00, 4 September 2008 (UTC)[reply]
Mondegreens seem to be cases where the two words/phrases sound very similar. That doesn't apply here. This wasn't caused by bad hearing, since all of us could make ourselves hear it as "Liverpool" with concious effort (and I don't think it was just suggestion, it took quite a bit of effort and there was a real "oh, wow! So it does!" moment when you did it, just like with optical illusions). It was said in a kind of sing-song voice, which may make the words sound a little more alike. --Tango (talk) 12:57, 4 September 2008 (UTC)[reply]
There has been some studies on the mis-perception of words. The McGurk effect is one example. Your example is probably most similar to Diana Deutsch's "Phantom Words" (an example is at [3] From that page: "two words, or a single word that is composed of two syllables ... are repeated over and over again. ... the first sound is coming from the loudspeaker on the left, the second sound is coming from the loudspeaker on the right; and vice versa. ... The words coming from the different spatial locations are offset from each other in time. As a result, listeners are given a palette of sounds from which to choose, and so can create in their minds many different combinations of sounds. After continuous exposure to these repeating words, listeners begin to 'hear' words and phrases that are not really there.") -- 128.104.112.147 (talk) 18:47, 4 September 2008 (UTC)[reply]
Cognitive psychologists at one time called this the "Gilbert and Sullivan Effect," as in the observation that if you know what the singers are jabbering in a patter song, you can then recognize the words, but without advance knowledge it may seem like gibberish. Edison (talk) 18:54, 4 September 2008 (UTC)[reply]
That toy must have a very, very poor speaker. Phonetically, the only sound shared by the two words is /ɚ/. Otherwise, they have a different syllable count, and the last two vowels of emergency, /ɛ/ and /i/, are so different from the /u/ in Liverpool in terms of articulation. The human brain is "wired" to try and make sense of any string of sounds as language, so I guess this case is more like straining your brain to make sense poor-quality audio, like when people in a stadium misunderstand a PA system with horrible acoustics.--El aprendelenguas (talk) 01:10, 5 September 2008 (UTC)[reply]
The different syllable count might be accounted for by the fact that it was in a sing-song voice. The fact that the words are made up of completely different sounds is harder to explain away! I wonder if they still have the toy, it might be interesting to do some studies. It seems it isn't a known phenomenon (I thought maybe there was some kind of duality between certain sounds and actually "li" and "em" are in some weird way connected, but apparently not [I wouldn't have believed it if I hadn't heard the toy!]). --Tango (talk) 01:30, 5 September 2008 (UTC)[reply]
You may be interested in Vowel#Articulation, especially the diagram of the articulation of vowels. Vowels that are located closer on the diagram are similar (if they share the same quality of roundedness). In fact, the first vowel sound in those two words (emergency, Liverpool) is sometime realized as the same vowel sound [ɪ]. Other sounds like [v] and [m] in English are not mistaken for each other except, unfortunately, in toys with poorly-crafted sound systems.--El aprendelenguas (talk) 01:59, 5 September 2008 (UTC)[reply]

I wonder if this is related to the need for people to guess what a speaker is saying when their articulation isn't clear. People with hearing issues would be particularly prone to this. For example, I occasionally watch NCIS, and whenever Pauley Perrette says anything, I have the greatest difficulty in deciphering just what she said (I do have a hearing loss, though). She has that kind of "rapid-fire squeaky nasal little-kid voice" that's becoming so prevalent among a certain demographic (girls, mainly). That makes it hard for a lot of people to know what they're saying even in ideal listening environments, but when background music is added to the mix (and NCIS has background music playing for virtually 100% of the program), it really does come down to guesswork. (If that reads like a bit like a rant, it wasn't meant to be). -- JackofOz (talk) 01:18, 7 September 2008 (UTC)[reply]

Looking for unbiased website to compare US presidential candidate's positions

As the subject says, I'm looking for an unbiased website to compare the US presidential candidate's positions on the issues. Any suggestions? I'm looking for the sort of thing that could be given to both (open-minded, uncommitted) Liberals and Conservatives as an antidote to the spin, something that is clear and straightforward and not with a particular axe to grind. --98.217.8.46 (talk) 12:17, 4 September 2008 (UTC)[reply]

Have you checked out our articles? Barack Obama and John McCain. I'm not sure they have exactly what you're after, but they'll give you some idea. --Tango (talk) 13:06, 4 September 2008 (UTC)[reply]
Better yet, we have an article that looks like exactly what you're after: Comparison of United States presidential candidates, 2008! --Tango (talk) 13:07, 4 September 2008 (UTC)[reply]
This site seems to be really good. It quotes from the candidates themselves with a minimum of editorializing. --Sean 14:45, 4 September 2008 (UTC)[reply]
The political compass is a wonderful website that shows the 'position' of the main candidates (though we now know who these are) on the scale. You can take the test and find where you fall too. It's pretty interesting and it'll show you wish you align to more closely based on your responses. It takes about 15 mins to complete though and asks some pretty tough questions! Anyhoo I expect they will update to have a comparison for the election itself once things get underway. 194.221.133.226 (talk) 15:38, 4 September 2008 (UTC)[reply]
I like On The Issues. It lists each candidate's (and pretty much everyone else in office) position on a slew of different topics, backed up by either their voting record or a direct quote. -- MacAddct  1984 (talk • contribs) 19:00, 4 September 2008 (UTC)[reply]
Personally, I go to the League of Women Voters (lwv.org). Unlike other women's organizations, they seem to me to have no political agenda per se. The site for California is http://www.ca.lwv.org/. --KNHaw (talk) 19:08, 4 September 2008 (UTC)[reply]

I did an online IQ test at http://www.iqtest.com

And I got a score of 132, apparently putting me in the lower end of the category "Gifted (2.3% of test takers)". I'd just like to know how reliable people think online IQ tests are, and to what extent that score can be considered accurate (it was a relatively short test with questions answered in a 'true or false' format). Would this possibly be better? The thing is, that many people on a forum that also took the test generally received high scores as well, and we can't all be that smart... can we?--Lost in kyoto (talk) 13:45, 4 September 2008 (UTC)[reply]

The true "IQ test" is "are you willing to pay $10 for an IQ score from some web page?" If you truly want to know your IQ, look up your local MENSA group. They take IQ tests seriously and will happily give you a real one. -- kainaw 13:51, 4 September 2008 (UTC)[reply]
I didn't pay any money. It emails you a score without a fee, whereas you only pay to see an 'in-depth profile', which I never considered going for as I felt it would be a waste of money.--Lost in kyoto (talk) 13:57, 4 September 2008 (UTC)[reply]
Don't forget the ton of SPAM you will also get, now that they have your email address for their 'free' service. They have to make money somehow, mate.--ChokinBako (talk) 14:04, 4 September 2008 (UTC)[reply]
It's OK; I just used a throwaway hotmail account for it which is already full of spam.--Lost in kyoto (talk) 14:07, 4 September 2008 (UTC)[reply]
I doubt it is a 'throwaway account'. Hotmail does not provide that service. They only have regular accounts. Actual 'throwaway accounts' are email accounts that expire, typically after 15 minutes. I seriously doubt you have got 5GB of SPAM in your hotmail account in 15 minutes. That would be a world record.--ChokinBako (talk) 14:20, 4 September 2008 (UTC)[reply]
Throwaway accounts are accounts that can be "thrown away" after use - that is, they are used for a specific purpose then forgotten about. To say it is "full of spam" does not imply that it has 5GB of spam in it, just that it has a lot of spam in it. If you have any further difficulties understanding the English language, I suggest you try the Language desk. DuncanHill (talk) 14:24, 4 September 2008 (UTC)[reply]
Sorry, Duncan, I was referring to the term 'throwaway' account as generally used on websites that advertise this service. The '5GB' I was referring to was the 5GB that hotmail gives (unless that has changed). I do realise that 'full of' something can also mean 'having a lot of XXX in it', but it does typically mean 'having far more than I would like to bother with/can handle, etc.' Try the language desk yourself, mate. We have lots of fun and games there.--ChokinBako (talk) 14:34, 4 September 2008 (UTC)[reply]
"Saucer of milk for table 5!" Meow, lads. :-) Getting back on topic. The questions are, a) Are these tests reliable? I have taken numerous web-based IQ tests since they first appeared on the Internets and they have all agreed a score to within about 5 points. So we can conclude that either they are accurate or they are all equally flawed (they could have motivation to inflate scores so people would pay for the trophy certificates they offer, but it is odd that they all, being quite different , agree). b) Can we all be that smart? Yes you could be. There could be a bit of selection bias at work. People likely to try the test probably have an inkling that they are going to do well. If we were to offer a gruelling fitness test to people, my guess is that we would get those people who already knew they were fit as participants rather than your average couch potato. Fribbler (talk) 15:23, 4 September 2008 (UTC)[reply]

What percentage of British people would know the difference between Slovakia and Slovenia?

What would you estimate? I'd say it'd be fairly low. If ignorance is widespread enough to allow for things like the grocer's apostrophe to be in common use, then I doubt that many people would know the difference between these two based on their similar-sounding names and relative proximity in location.--Lost in kyoto (talk) 13:49, 4 September 2008 (UTC)[reply]

Quick check and I found no studies on this. Please keep in mind that this is a Reference Desk, not a What is your opinion on something I just thought about Desk. If you want to have a discussion, use one of the many (MANY) online discussion forums. -- kainaw 13:53, 4 September 2008 (UTC)[reply]

I think the majority of educated British who have some sort of reason to know would know the difference. After all, there was never such a country as 'Czechoslovenia' and we have just spent a decade fighting two wars in the Balkans. People who don't know don't care, just like most Brits don't need to know the difference between Japan and China. --ChokinBako (talk) 13:55, 4 September 2008 (UTC)[reply]

I find that very hard to believe. Not knowing the difference between Slovakia and Slovenia, maybe, but most people know the difference between Japan and China! --Cameron* 14:20, 4 September 2008 (UTC)[reply]
Well, my mum didn't know the difference and she has a degree.--Lost in kyoto (talk) 13:59, 4 September 2008 (UTC)[reply]
Also, Slovenia is the only EU member from the former Yugslavia and seems the most progressive, although Croatia seems to be catching up as well. It's only further southeast that the major wars in the Balkans were fought.--Lost in kyoto (talk) 14:09, 4 September 2008 (UTC)[reply]
Croatia is next door. Bosnia is next to that, then Serbia. Not very far away, and Slovakia Slovenia (whoops! I made the very mistake we speak of!) was in the news during that time, anyway. So was little Macedonia and all the rest. I don't see the relevance of your answer to mine, though. It sounds more like a complaint against your mum.--ChokinBako (talk) 14:17, 4 September 2008 (UTC)[reply]
Here in the United States, many people I talked to did not realize that Russia had invaded Georgia, the country, not Georgia, the state in the US... -- MacAddct  1984 (talk • contribs) 19:11, 4 September 2008 (UTC)[reply]
Ha! No wonder the U.S. rallied around Georgia. (Oh, and maybe some lingering feelings from the Cold War.)Twas Now ( talkcontribse-mail ) 19:43, 4 September 2008 (UTC)[reply]
What? They actually thought that Georgia (state) includes an area called "South Ossetia"? -- JackofOz (talk) 21:46, 4 September 2008 (UTC)[reply]
Moreover, a separatist region called South Ossetia - I can just imagine parts of US states declaring independence and their citizens all getting Russian passports! Now, that would be worth watching American news channels for! --Tango (talk) 21:56, 4 September 2008 (UTC)[reply]
What counts as knowing the difference? Just knowing that they are two different countries? I would expect most people know that - if it has a different name, it's a different country (Myanmar and Burma aside!). Actually knowing anything about either country is probably more rare. --Tango (talk) 21:56, 4 September 2008 (UTC)[reply]
When Argentina invaded the Falkland Islands, most people in the UK thought they had invaded a part of Scotland. We couldn't get our heads round why they thought it was part of their territory!--ChokinBako (talk) 23:02, 4 September 2008 (UTC)[reply]
Really? I knew where they were, and don't recall any of my schoolmates suffering from that confusion. DuncanHill (talk) 23:09, 4 September 2008 (UTC)[reply]

whilst i take your point about the grocer's apostrophe, its maybe a little unkind to then assume all my fellow compatriots are brainless dum-dums. At the risk of coming across all 'henry higgins', I would imagine that despite our national inability to speak our own language (or anyone else's for that matter) we're quite good at world geography, having conquered most of it as some stage or another.. —Preceding unsigned comment added by 82.22.4.63 (talk) 17:45, 7 September 2008 (UTC)[reply]

Why are Hungary, Finland and Estonia anomalous in Europe as well as Romania?

The former three have Finno-Ugric languages, which are not part of the Indo-European language family. This in itself is anomalous for Europe, but even more strange is the fact that these languages are found in Northern Europe and then in a small pocket in Central Europe, with areas of Slavic and Germanic tongues seperating them. Also, why is Romanian a Latin language when it is largely surrounded by Slavic ones? How can these enclaves be explained? Also why are the people of a nation considered to be representatives of their language families? There are Germanic people, Latin people and Slavic people, but this doesn't seem to be a worthy categorisation of people, just as white people and black people are in my opinion crude and outdated Americanisms which would be better replaced with European people and African people (Sub-Saharan).--Lost in kyoto (talk) 13:53, 4 September 2008 (UTC)[reply]

Finland and Estonia are right next to Russia, which has literally hundreds of Fenno-Ugric languages in the area near to these countries. Romania is not very far from Italy. Hungary may seem isolated, but then that may explain why its language is so different from other Fenno-Ugric languages.--ChokinBako (talk) 14:00, 4 September 2008 (UTC)[reply]
Simple answer: People move. For more information, see History of Europe. As for describing people by the language group they speak - it is merely a characterization of the person, not a judgement. For example, saying that a guy is wearing brown shoes is a characterization used to identify him. It is not (necessarily) a judgement about him. -- kainaw 13:57, 4 September 2008 (UTC)[reply]

The Finnic languages in the general Baltic area have more or less remained in place as far back as we can trace with any certainty. However, Hungarian was imported from hundreds of miles to the east in the Hungarian migrations of ca. 900 A.D. Also, there are some enigmas about the survival of Rumanian -- the main area of Rumanian was presumably generally north of the Jireček Line, yet it's far from clear where in general Balkans region a substantial Romance-speaking society could have been hiding out during the 7th-century A.D. and for several turbulent centuries thereafter -- when the Slavs had overrun the western Balkans as far south as the Peloponnese, while the eastern Balkans were initially split between Byzantines and Avars, and then suffered several further waves of invasions from Bulgars and Magyars (Hungarians)... We have a whole article on Origin_of_the_Romanians...AnonMoos (talk)

Romania was, at least, part of the Roman Empire, but as Kainaw says, people migrate. The whole of the Balkans and Central Europe are a linguistic jigsaw puzzle, and you can only understand it by looking at the way one wave of migration and conquest has followed another all through the region's history. On a larger scale, the same must be true of the whole world. Strawless (talk) 17:37, 11 September 2008 (UTC)[reply]
Romania was a frontier during the latter part of the Roman Empire (circa roughly 300 to 400 AD). It was a wild and unsettled area, yet it technically belonged within the Empire. To address this problem, as well as to solve the problem of rewarding lands to retiring Roman Army soldier (after putting in their allotted time), the Empire gave away tracts of land to these newly made Roman citizens (upon leaving the Army, Roman soldiers were also made citizens). Most of the men in the Roman Army at this time were from far-scattered lands throughout the vast Roman Empire and beyond. The language they all had in common was Latin, but it was a rather earthy, vulgate form of Latin. The soldiers would learn this "Creole" Latin as a second language, for the most part. So, when they retired to their farms in the new settlements being created in Romania, it was the "Army Latin" that was spoken the most. From this foundation we have the beginnings of the modern language of Romanian. I have been told (although this is unfortunately for me unverified) that of all the Romance Languages (Italian, Spanish, French, etc) that Romanian is the closest to what Latin would have sounded like, as it was spoken by the average person on the streets of Rome. Saukkomies 15:29, 11 September 2008 (UTC)[reply]

Spies?

I remember hearing this story, possibly on the radio, about some Americans that by some strange turns of events ended up spying for the Japanese in WW2, but I can't find anything more about it now.

Here's what I remember of the story. A couple of American men go to Mexico where they end up with money troubles and have to sell their car, but then they're stopped when trying to come back to the US because the border guards want to either see their car or have some record of its sale/destruction. This causes them to be stuck in Mexico, but at some point they hop a boat to Japan shortly before Japan's involvement in WW2. Now in Japan and citizens of the enemy, they are convinced by the Japanese to go back to the US and spy. When the guys reach the US shore they turn themselves in, but are still tried for spying anyways.

I've tried to search for more about this, but I've got nothing. Laenir (talk) 16:15, 4 September 2008 (UTC)[reply]

I don't have an answer, but we do have a List of Japanese spies, 1930–45. Unfortunately none of them seem to match your description. --LarryMac | Talk 17:26, 4 September 2008 (UTC)[reply]
Several German-American spies invaded the US via submarine in WW2 as part of Operation Pastorius and were captured when some of them turned the others in. They were tried and most were executed after appeals to the Supreme Court in Ex parte Quirin. Herbert Haupt had the adventure you speak of, and got executed by the US government. He made their way from the U.S to Japan to Germany, was trained with the others as a saboteur and sent to the US via U-boat. Edison (talk) 18:49, 4 September 2008 (UTC)[reply]


I'm sure that's it. Thanks, Edison! 206.83.160.121 (talk) 20:32, 4 September 2008 (UTC)[reply]
Wrong link. Correct is Herbert Hans Haupt. --Cookatoo.ergo.ZooM (talk) 20:51, 4 September 2008 (UTC)[reply]

France, England and classical music

Considering the cultural, financial and military importance of France and England for the last 400 years or so, it startles me that there are so few reknowned French and English composers and (as far as I am aware) almost no operas written in those languages. Is there a historical/cultural explanation for the domination of music by Italy, Russia and the Germanic nations? Thanks 90.192.223.228 (talk) 17:43, 4 September 2008 (UTC)[reply]

Stephen Jay Gould wrote an essay that touched on what he termed (iirc) "The German Composer Disease". That might not be quite it, but it's close. Basically, the question becomes, "Why is it that, a few notable exceptions aside, every well-known piece of "classical" music was composed by a German speaker between the years 1700 and 1900?" There will be short term explanations that deal with funding, etc. but it still seems odd, and he didn't come up with any kind of compelling answer that I can recall. In a way, I suppose the question makes as much sense as asking why German speakers between 1700 and 1900 produced very few hard rock performers. Matt Deres (talk) 18:52, 4 September 2008 (UTC)[reply]
Do you remember the essay title or the book where you read it?--droptone (talk) 19:49, 4 September 2008 (UTC)[reply]
I'll take a look, but I must have a dozen of his books kicking around. If I can't find it reasonably quickly, I'll drop you a line at your talk page. Matt Deres (talk) 13:53, 6 September 2008 (UTC)[reply]
Thanks a bunch!--droptone (talk) 12:46, 10 September 2008 (UTC)[reply]
French opera spans the centuries between Lully and Poulenc. Giacomo Meyerbeer was actually a French composer. Chamber music by Franck, Debussy, Fauré, Ravel is central to the repertory. Not all music is orchestral music for the big concert hall. --Wetman (talk) 19:40, 4 September 2008 (UTC)[reply]
If we're talking about operas written between 1700 and 1900, I wouldn't regard the enormous mass of operas written by Italian composers as "a few notable exceptions". (By the way, questioner, the 2nd syllable of "renowned" is pronounced like "now" - not like "know" - and is spelled accordingly. I wouldn't normally nitpick, but this is becoming an increasingly common error, so others may benefit from this comment.) -- JackofOz (talk) 21:27, 4 September 2008 (UTC)[reply]
Nor would I, but I don't know of any classification system where opera and classical/romantic music are combined in such a way. Everyone knows opera is the one with the big lady wearing steel lingerie and classical music is one with the shaggy-haired conductor waving a baton. ;-). Matt Deres (talk) 13:56, 6 September 2008 (UTC)[reply]
Harold C. Schonberg’s answer (which should be taken with a grain of salt) to the question of why so few British composers emerged until the twentyish century “British School” of composition arose, is that Handel literally chilled British innovation in composition for hundreds of years by giving the English a sever obsession with choral music, particularly sacred choral music.
Schools of composition move in cycles. Someone somewhere will come up with something pretty neat and everyone will rush to try the new style. The French and British have had some extremely famous composers, mostly in the 19th and 20th centuries. They are perhaps not as familiar to you because there is currently a strange obsession with playing the very old music ad nauseam.
For French composers there are the composers named by Wetman above. Add to that Olivier Messiaen, Edgard Varèse, Francis Poulenc, Darius Milhaud, Arthur Honegger, Finally, of very special note, there is Nadia Boulanger who is known as perhaps the greatest and most famous composition teacher of all time; a woman who guided the development of American music itself and taught nearly all the greatest American composers for half a decade. (See List of French composers for many more names).
For famous British composers there is Benjamin Britten, Ralph Vaughan Williams, Edward Elgar, Peter Maxwell Davies, Michael Tippett, and of course Arthur Sullivan (See List of British classical composers for many more names). Hope this helps. --S.dedalus (talk) 04:17, 5 September 2008 (UTC)[reply]

Which books did Sarah Palin seek to ban from a local library?

Willy turner (talk) 18:25, 4 September 2008 (UTC)[reply]

This page has a link to an alleged list of books that Palin wanted to ban, but even on the original page with the link, there's a great deal of skepticism as to whether the list is accurate or not. It is true that Palin tried to get the head librarian at the Wasilla libary fired for "not giving full support to the mayor": [4]. Corvus cornixtalk 18:38, 4 September 2008 (UTC)[reply]
The list is not accurate at all. Please don't get confused by it and spread misinformation—it makes everyone look bad.
It's not clear that Palin ever gave the librarian a list of books—from the articles it sounds like she just sounded out the idea of getting some books removed. --98.217.8.46 (talk) 23:03, 4 September 2008 (UTC)[reply]
I agree that the coverage so far indicates no list was produced, but you might be interested in the American Library Association's list of 100 most frequently challenged books from 1990-2000, which is probably quite representative of the sorts of books people don't want others to read: [5]. I was going to go see if "Daddy's Roommate" was available at the Wasilla public library, but their server is down. :) --Sean 14:36, 5 September 2008 (UTC)[reply]
It's back up now, and SCORE! :D --Sean 17:27, 5 September 2008 (UTC)[reply]
A very recent Time Magazine article on Palin briefly mentioned this happening, though there were only one or two sentences about that. Apparently, the censorship was intended to satisfy people wanting to avoid swearing in books. Vltava 68 (talk, contribs) 09:22, 8 September 2008 (UTC)[reply]
According to factcheck.org, Palin never tried to have any books banned, and the phony list in circulation contains books that hadn't even been published at the time. 24.210.252.153 (talk) 05:55, 10 September 2008 (UTC)[reply]

When British police take the cars of people charged with motoring offences; why dont they sell them rather than crush them?

Willy turner (talk) 18:25, 4 September 2008 (UTC)[reply]

{wild speculation) I would imagine that it is because the police are not allowed to make a profit from these types of crimes, also by crushing the car they are providing a deterrent to further crime whilst also removing what can be a dangerous vehicle from the streets (positing that uninsured and untaxed cars may be damaged/unsafe). They are allowed to liquidate the assets of someone to recover the value of any fine or damages assessed by a court via intermediaries but only in certain cases. They have no objection to selling cars from other crimes or even recovered stolen goods (in fact they have their own auction website for this at http://www.bumblebeeauctions.co.uk ). Nanonic (talk) 19:09, 4 September 2008 (UTC)[reply]
Presumably they don't have their own crushing facility but use a commercial one. That suggests that someone is profiting from crime. -- JackofOz (talk) 21:20, 4 September 2008 (UTC)[reply]
Police officers, lawyers, gaolers, newspaper proprietors, burglar-alarm salesmen, they all profit from crime. DuncanHill (talk) 21:25, 4 September 2008 (UTC)[reply]
You missed out "criminals"! ;) --Tango (talk) 21:57, 4 September 2008 (UTC)[reply]
No, I mentioned newspaper proprietors. DuncanHill (talk) 22:00, 4 September 2008 (UTC)[reply]
What we have here is a fundamental structural flaw. It's just so unfair that anyone can profit from crime, except the perpetrator. Without the perpetrator, all the others would be out of a job, or at least have a severely reduced income.  :) -- JackofOz (talk) 22:02, 4 September 2008 (UTC)[reply]
Maybe criminals should unionize to secure for themselves the true value of their labour. DuncanHill (talk) 22:06, 4 September 2008 (UTC)[reply]

Meanwhile, getting back to the tedious business of answering the question, I read in Hansard for 8 Nov 2005 a reply from Alistair Darling, the then Transport Secretary, who said "and the police now have the power to crush or sell cars that are not claimed or insured within two weeks." I guess crushing sounds more compelling in HMG's advertising. From a 19 Jul 2005 debate in the house, this time on the subject of annoying moto bikes, it is suggested that "there is a view locally that if they have been used illegally, they should not only be confiscated but crushed and taken out of the market so that the young people who are buying them might begin to focus on just how serious offence they may have committed". The Select Committee on Public Accounts Twenty-Eighth Report mentions DVLA crushing operations in 2002/3, commented that "The Agency conceded that, in the main, it is the very oldest vehicles that are wheel-clamped which go unreclaimed by the keeper, and which ultimately have to be crushed at the Agency's expense", and one can see why the powers that be may well think it a good thing to get old bangers out of the market entirely rather than, to be cynical, being sold at auction for £50 to exactly the sort of person who would then go out and drive it uninsured again. --Tagishsimon (talk) 22:25, 4 September 2008 (UTC)[reply]


I've seen and heard of Police auction where the police sell recovered items that haven't been 'claimed'. In my local area you can go along to these 'auctions' and buy yourself a bicycle for (sometimes) bargain prices. I would be surprised if similar things didn't occur for motorised vehicles too. 194.221.133.226 (talk) 08:11, 5 September 2008 (UTC)[reply]
In the US such auctions are necesitated by the shear number of vehicles of all sort which are impounded, however, nowa days inthe town where I live the task of auctioning has been tunred of to an online auction house that psecializes in practically fraud since you can not touch or feel and returns are not allowed sorry. In other words if you can't touch or feel then forget it. The local governments then dispose of property periodically that does not sell even though the administrations are in need of cash. However the real reason cars are crushed in the UK is because cars are thought of there as living agents fully capable of causign there owner, whomever it might be. to engadge in crime. These cars are bad card and must be estroyed in order to rid the UK of such agents of the Devil. Trust me, I know I am right on this. —Preceding unsigned comment added by 71.100.2.227 (talk) 10:35, 5 September 2008 (UTC)[reply]
Is that a Deodand? Llamabr (talk) 19:04, 5 September 2008 (UTC)[reply]

Reliability of CliffsNotes, etc

It's sometimes argued in FAC discussions that CliffsNotes and similiar guides are not "reliable sources". I can understand why Cliffsnotes would be considered unscholarly or anti-itellectual, but are they really unreliable? Can anyone give a specific example of something included in a CliffsNotes guide that is incorrect, or even just misleading? Zagalejo^^^ 18:49, 4 September 2008 (UTC)[reply]

I'm sure there are plenty of factual errors in CliffsNotes, however, that alone would not make them not WP:RS. Rather, and this is my HO, the summary nature of these notes is what limits them as a source. An encyclopedia should contain all acceptable viewpoints, not just the ones that will get you through the midterms! --Regents Park (count the magpies) 20:40, 4 September 2008 (UTC)[reply]
I don't think I agree if you're saying that CliffsNotes & similar are more prone to factual errors than the average academic journal. I'd say the opposite, if anything, and I'll make that case more fully here if anyone really wants me to. Like the original poster, I tend to resist the thinking that books like that are not WP:RS. Sometimes when you are trying to source the basics about a topic, doing so from a book designed to discuss the basics is using the best source. AndyJones (talk) 20:49, 4 September 2008 (UTC)[reply]
I'm not saying that they are more prone to factual errors. Rather that factual errors are not the point. The summary nature of cliffnotes and the fact that they are not necessarily assembled by established scholars in the field, makes them less than reliable for anything other than main ideas. --Regents Park (count the magpies) 21:00, 4 September 2008 (UTC)[reply]
Well (a) we're a general-purpose encylopedia: the main ideas is what we do; and (b) although I don't have any CliffsNotes here, I've a number of York Notes, Cambridge Notes and Brodie's Notes and at a quick glance it seems to me they're all written by University lecturers. AndyJones (talk) 21:10, 4 September 2008 (UTC)[reply]
I'd argue that it has nothing to do with errors. It has to do with the fact that CliffNotes is not the original source (unless one is writing an article *about* CliffNotes). Wikiant (talk) 23:00, 4 September 2008 (UTC)[reply]
We're meant to use secondary sources, aren't we? DuncanHill (talk) 23:02, 4 September 2008 (UTC)[reply]
They're not the only acceptable sources, but they're the ones we mainly use. -- JackofOz (talk) 01:29, 5 September 2008 (UTC)[reply]
  • If you are looking for just any secondary source about a literary work, then CliffsNotes are a reliable source. But if you are trying to write a featured article about a literary work, then I would consider CliffsNotes to be a mediocre source and not worthy of use as a source in a featured article. Any literary work which has CliffsNotes of it probably has been the subject of scholarly commentary which is worthy of being cited in an encyclopedia, whereas CliffsNotes are commentary on the work aimed at high school students who either had difficulty understanding the work or didn't even read it. --Metropolitan90 (talk) 17:23, 6 September 2008 (UTC)[reply]
Thanks for the replies, everyone. I'm not specifically planning to use CliffsNotes as a source in an article; I was just wondering if anyone was aware of any bonafide factual errors in the guides. Zagalejo^^^ 07:55, 8 September 2008 (UTC)[reply]

Hamlet plot query

Plea for help for any Shakespeare experts... during the later stages of 'Hamlet', Hamlet is banished to England by Claudius. His ship is attacked by pirates, and Hamlet alone is captured. But why do Rosencrantz and Guildenstern continue on the ship to England? Their objective, given to them by Claudius, was to escort Hamlet to England and pass on the letter demanding Hamlet's execution to English authorities, etc. However, since their escorting duty has gone so badly wrong, surely the bumbling and timid pair, R&G, should return to Denmark for advice and further orders from Claudius. They have no need to go to England, and would probably have avoided their deaths in a 'real' situation simply by returning home. I have done research online and from books, and there is no answer. So why DO R&G bother to continue to England? Or have I missed something? 87.114.133.121 (talk) 19:35, 4 September 2008 (UTC)[reply]

What are they supposed to do, swim? Or somehow compel the captain of the ship to turn around? Corvus cornixtalk 19:50, 4 September 2008 (UTC)[reply]
I suspect the OP is under the impression that a private ship was hired for the sole purpose of taking Hamlet to England. This is extremely unlikely as three passengers, even with an entourage, would make a poor payload for the captain and/or the owners. ៛ Bielle (talk) 20:00, 4 September 2008 (UTC)[reply]
If that is the OP's impression I don't think there's anything in the text of the play which makes it demonstrably wrong. Kings command naval vessels, and can commandeer merchant vessels. Besides, OP's question is still a valid one if you phrase it as "why do they bother to visit the English King?" I could suggest a number of answers to that, of course. AndyJones (talk) 20:57, 4 September 2008 (UTC)[reply]

Okay, no problem. Thanks guys. 87.114.133.121 (talk) 20:35, 4 September 2008 (UTC)[reply]

Also, when you bumble a task given to you by a tyrannous king, it is best not to appear in front of said king for a (long) while. --Regents Park (count the magpies) 20:37, 4 September 2008 (UTC)[reply]
Keep in mind that Hamlet is a work of fiction. There wasn't any prince; there wasn't any ship; there weren't any pirates; Rosenkrantz and Guildenstern didn't go anywhere because they were not in fact real people. Shakespeare has all kinds of inconsistencies and loose ends that he fails to tie up. In any case, in Act IV Claudius has R & G take Hamlet out of the country purportedly to protect him from the consequences of killing Polonius, though mostly because Hamlet is "loved of the distracted multitude." Claudius's letter to the king of England was sealed; R & G's orders were to deliver the letter. Hamlet found it, read it, and replaced it with his own version requesting England
...That, on the view and knowing of these contents,
Without debatement further, more or less,
He should the bearers put to sudden death,
Not shriving-time allow'd.
--- OtherDave (talk) 11:27, 5 September 2008 (UTC)[reply]
Oh, don't be such a spoilsport :-) Of course it's fictional; it's our job, as fans, to to explain inconsistencies, not just complain about them or push them aside. That's what real fan fiction should be, not just stuff for people who only want to write about sex.
Now, you could pull a "The reference desk is not for speculation" and that would be fine; it does call for a bit of speculation here, because Shakespeare doesn't tell us, since they're not consequential tot he plot. However, since inquiring minds want to know, I would suggest that Claudius may well have sent them because they were known for bungling things, anyway; sort of like Laurel and Hardy. I mean, as a tyrant, you're not going to send your most trusted people there. You're going to send red shirted ensigns, which Rosencrantz and Guildenstern were the first of. (Star Trek reference - surprised there's not a page for that.) If Claudius did call them "trusted," then perhaps it is in a satirical way. —Preceding unsigned comment added by DTF955 (talkcontribs) 16:35, 5 September 2008 (UTC)[reply]
Ah, but was it really a work of fiction? The Oxfordians among us would argue differently. Various parts of Hamlet, such as the encounter with Fortinbras's army and Hamlet's brush with buccaneers, do not appear in any of the play's sources, to the puzzlement of numerous literary critics. These might be explained by reference to actual events in the known life of Edward de Vere, 17th Earl of Oxford. Just as Hamlet's review of Fortinbras's troops leads directly to an ocean voyage overtaken by pirates, de Vere's meeting with the German duke Jan Casimir (who was leading troops towards Paris) was followed by a Channel crossing intercepted by pirates. He had met Jan Casimir on his way back from Italy (his known travels in that country are all exactly mirrored in references in the plays; Shakespeare is not known to have ever left England). In April 1576, de Vere boarded a ship in order to cross the Channel back to England. The ship was attacked by pirates, his luggage was ransacked, and he was stripped naked. They were allowed to continue on their way to England, however. But he had another experience with pirates. In 1585, he sent a boatload of apparel, money, wine and venison back to England from the Lowlands. The ship was intercepted by Spanish pirates off Dunkirk and was looted. The pirates discovered a letter to de Vere from Lord Burghley appointing him commander of the horse. De Vere transformed this 2nd experience into something useful: Hamlet contains not only an encounter with pirates but also an analogous plot twist involving suborned letters at sea. (I've quoted liberally from Mark Anderson: Shakespeare by Another Name.) If one sees these plays as an ongoing coded autobiography of de Vere as much as pure fictional entertainment, many of the loose ends and unsourced references make sense. Then, you don't have to worry too much about the apparent illogicality of R&G continuing on to England. -- JackofOz (talk) 22:34, 5 September 2008 (UTC)[reply]


September 5

Vaudeville hat

Does anybody know the actual name of the distinctive round, flat-topped hat worn by vaudeville performers?

Example: http://www.pioneerdrama.com/titlegraphics/vaudeville-hat-c(1).gif

Also, would anybody happen to know of any reputable dealers of said style of hat? Not this costume stuff, real legitimate hat. Thank you!! Kenjibeast (talk) 04:14, 5 September 2008 (UTC)[reply]

Searching for "straw hat" led me to the boater article. --Kjoonlee 04:32, 5 September 2008 (UTC)[reply]
But that's not right. Pork pie hat looks closer. --Kjoonlee 04:48, 5 September 2008 (UTC)[reply]
It is a boater. 194.50.118.230 (talk) 08:55, 5 September 2008 (UTC)[reply]
Yes, a boater is commonly worn by such performers, including traditional barbershop quartets.Gwinva (talk) 09:56, 5 September 2008 (UTC)[reply]
OP here. http://en.wikipedia.org/wiki/Image:Porkpie.jpg <--- that is the exact model of hat I'm looking for. So is that a pork pie or a boater? What exactly is the difference? —Preceding unsigned comment added by Kenjibeast (talkcontribs) 20:03, 5 September 2008 (UTC)[reply]
The most obvious difference is that a porkpie hat is made of felt, while a boater is made of straw. --LarryMac | Talk 20:07, 5 September 2008 (UTC)[reply]

BTW, you can link to images by using [[:Image:Porkpie.jpg]]: Image:Porkpie.jpg --Kjoonlee 02:42, 6 September 2008 (UTC)[reply]

I'm sure they are not the only source, but a quick Google of "traditional straw boater" found me this hatter on London's Jermyn Street which seems to be a long-established dealer selling the genuine article. Karenjc 12:30, 6 September 2008 (UTC)[reply]

When I think of vaudeville and hats, the Susquehanna Hat Company comes to mind. This was an (Abbott and Costello routine,[6] Youtube link]] wherein many straw boaters get destroyed one after another. A fist punches so nicely through the top of one. Edison (talk) 19:26, 6 September 2008 (UTC)[reply]

The boater that I used to wear when punting in Cambridge indeed looked just like the second picture in the boater article. But what that article does not mention is that it was heavily lacquered, so that it was rigid and fairly heavy. You would not find it easy to punch the crown out of one of those! --ColinFine (talk) 20:04, 8 September 2008 (UTC)[reply]

Czechoslovak hijacking

I read in Petr Sis's book The Wall that there was a hijacking of a plane to West Germany on June 8, 1972, by a bunch of Czechoslovaks. Unfortunately, I could not find any more information. Vltava 68 (talk, contribs) 09:26, 5 September 2008 (UTC)[reply]

This is mentioned briefly at 1972#June and here. — Twas Now ( talkcontribse-mail ) 09:36, 5 September 2008 (UTC)[reply]

Limitation on electing Presidents?

This is something I remember reading (at some point in my life, not recently) but I can't find it anywhere now. There is, or was, or was being considered, a rule/law to the effect of "Two people from the same State may not be elected President of the United States in successive terms." So (for example) Gov. Rick Perry of Texas legally couldn't be elected President in 2008 because he's from the same state as George W. Bush.

It isn't in President_of_the_United_States#Qualification.2C_disqualification_and_common_practice, so it's clear to me that the aforesaid rule/law is not the case, at least not now. As I (incorrectly) recall, it was in the U.S. Constitution; but I'm wrong about that. The closest I find in the Constitution is part of the 12th Amendment which requires electors to vote separately for President and VP, at least one of which "shall not be an inhabitant of the same state as themselves."

Any idea what I'm talking about? (If this isn't the proper place to ask this type of question, please point me in the right direction.) Thank you.Fitfatfighter (talk) 09:58, 5 September 2008 (UTC)[reply]

It's a confusion between candidates and electors. See this article from Snopes.com. Article II requires that at least one of the persons chosen by a state's electors "shall not be an inhabitant of the same state with themselves." In other words, if McCain and Palin were from the same state, under the constitution no more than half of any state's electors (not voters, mind you) could choose them. --- OtherDave (talk) 11:36, 5 September 2008 (UTC)[reply]
I don't think your explanation is correct. If McCain and Palin were from the same state (say, Arizona), then electors from Arizona only (not "any state") would be able to vote for only one or the other. This is really only an issue if you're looking at either a close election or a state with a high electoral vote. Were McCain and Palin both from Arizona, the Republicans would be sacrificing 5 EVs (10/2) for the pair, or 10 for Palin if they were willing to risk a Dem VP. Were they both from Alaska, it'd be a 1-2 EV drop, which might be entirely acceptable -- if you can win by even 5, who cares if you give up 1 or 2. If they were both from California, having to split those 55 votes would likely be too high a price to pay. This precise scenario is why Dick Cheney is currently a resident of Wyoming -- splitting Texas' ~35 EVs in 2000 was a ludicrously high price and would have cost the Republicans the election. — Lomn 14:17, 5 September 2008 (UTC)[reply]
You're right, I misstated. Article II says "The Electors shall meet in their respective States, and vote by Ballot for two Persons, of whom one at least shall not be an Inhabitant of the same State with themselves." You could in theory run two candidates from Alaska (assuming you could find a second person with the outstanding qualifications of having been a small-town mayor and a moose hunter) and throw away the whole state's electoral vote.
The main point was to clarify the meaning of Article II. --- OtherDave (talk) 15:45, 5 September 2008 (UTC)[reply]
If Alaska is out of moose-hunting mayors, perhaps they have some "community organizers"—apparently that qualifies one for high office these days. ;-) —Kevin Myers 16:10, 5 September 2008 (UTC)[reply]
Of course, the "same state" requirement is pretty much a dead letter in these days of high mobility. If a candidate for president picks a candidate for vice-president who happens to live in the same state, one of them can just move to another state. --Anonymous, 01:13 UTC, September 6, 2008.

Had there actually been a rule in the Constitution that "Two people from the same State may not be elected President of the United States in successive terms", the first person to suffer from that rule would have been the "Father of the Constitution" himself! Early in American history New Englanders were concerned about the dominance of the Virginia Dynasty, and in the Hartford Convention proposed such a rule. This was perhaps the rule (though never adopted) that the original questioner was thinking of. —Kevin Myers 15:59, 5 September 2008 (UTC)[reply]

It looks like the Hartford Convention/Virginia Dynasty thing is what I was talking about. Thank you Kevin (and everyone else).Fitfatfighter (talk) 21:14, 5 September 2008 (UTC)[reply]

I'm slightly intrigued by the wording of the amendment: "... one at least shall not be an Inhabitant of the same State with themselves". Is "the same ... with" a recognised construction? Normally, it would be "the same ... as", wouldn't it? Or is there some subtle legalistic gobbledygookesque nuance I'm not seeing? -- JackofOz (talk) 21:36, 5 September 2008 (UTC)[reply]

It does seem strange, but remember the amendment was written over 100 years ago, so the language might have changed slightly. The use of "with" does make some sense, you live in a state with another person. It's probably correct English, just uncommon these days. --Tango (talk) 23:34, 5 September 2008 (UTC)[reply]
It's mostly the change in the language, Jack; the twelfth amendment was ratified in 1804, so that's 200 years. And if you want some linguistic fun, try parsing the second amendment. The founders were a comma-happy bunch. --- OtherDave (talk) 02:29, 6 September 2008 (UTC)[reply]
Erk. The version with 3 commas is nonsense, basically. I feel sorry for you guys. -- JackofOz (talk) 04:38, 6 September 2008 (UTC)[reply]
Oh yeah, 2008-1804=204, not 104... I always say mathematicians can't do arithmetic... --Tango (talk) 04:57, 6 September 2008 (UTC)[reply]

pre-marital sex

How is 1.) premarital sex and 2.) pregnancy regarded and treated by various religions? —Preceding unsigned comment added by 71.100.2.227 (talk) 10:23, 5 September 2008 (UTC)[reply]

Premarital pregnancy, or just pregnancy? — Twas Now ( talkcontribse-mail ) 10:28, 5 September 2008 (UTC)[reply]
I meant to ask only about sex so both 1.) premarital sex without pregnancy, 2.) premarital sex with pregnancy but no birth prior to marriage, 3.) premarital sex with pregnancy and birth prior to marriage. —Preceding unsigned comment added by 71.100.2.227 (talk) 10:48, 5 September 2008 (UTC)[reply]
Sorry to bother you again, but your second one gets me—is there no birth before marriage because the mother had an abortion, or because the couple got married before the baby was born? Or would you like to know both cases, in which case there would be four cases:
  1. Premarital sex without pregnancy,
  2. Premarital sex with pregnancy and birth prior to marriage
  3. Premarital sex with pregnancy, followed by marriage, then childbirth
  4. Premarital sex with pregnancy, followed by an abortion,
Twas Now ( talkcontribse-mail ) 10:56, 5 September 2008 (UTC)[reply]
No bother, in fact,
  1. Premarital sex without pregnancy,
  2. Premarital sex with pregnancy and birth prior to marriage
  3. Premarital sex with pregnancy, followed by marriage, then childbirth
  4. Premarital sex with pregnancy, followed by an abortion,
  5. Premarital sex with pregnancy and birth prior to marriage, followed by adoption
  6. Premarital sex with pregnancy, followed by marriage, then childbirth, followed by adoption
—Preceding unsigned comment added by 71.100.2.227 (talk) 11:06, 5 September 2008 (UTC)[reply]
I was tempted to ask about paid premarital... but forget it.
And there are a few religions not too crazy about pregnancy -- such as the United Society of Believers -- though this is perhaps not the best strategy for passing on your beliefs to future generations. --- OtherDave (talk) 11:40, 5 September 2008 (UTC)[reply]
Yes, but they did believe in adoption as a viable alternative to reproduction themselves. —Preceding unsigned comment added by 71.100.2.227 (talk) 11:52, 5 September 2008 (UTC)[reply]
Did they take too much to heart the maxim "If your parents didn't have children, there's a good chance you won't either"? -- JackofOz (talk) 21:28, 5 September 2008 (UTC)[reply]
Don't forget about premarital pregnancy without sex, which is a topic of much import in some religions. -- 128.104.112.147 (talk) 19:38, 5 September 2008 (UTC)[reply]
This question doesn't appear to be well answered. I once saw a page that listed the different religions and who considered what a sin. Your answer:
  1. Islam considers all premarital sex to be sinful.
  2. Christianity almost always does.
  3. Judaism does not explicitly prohibit it, but generally hints that it's bad. explicitly prohibits it (Deuteronomy 22:21.
  4. Dharmic religions tend to be neutral on the issue.
  5. Animistic religions of course are too widespread to pin down.
  6. It's generally important to note the fact that marriage exists in every society, and religion often works to enforce morals. Thus, any religion, even if not specifically prohibiting it, has normally discouraged it in the past. Does this answer your question? Magog the Ogre (talk) 19:05, 5 September 2008 (UTC)[reply]
That still doesn't answer how it's treated. In the Baha'i Faith sex out of wedlock is forbidden, as is abortion as a form of birth control, but, other related parts of the particular circumstances aside, those actions are normally viewed as being between the individuals and God. Coreligionists close enough to them would probably offer support as friends and others (including the administration) would most likely stay out of it. I cannot speak for the other religions but I would guess that it largely depends on what the particular group of the religion is like and what the prevailing culture is like. - Lambajan 19:39, 5 September 2008 (UTC)[reply]
I thought maybe other people would weigh in with how this is treated. I looked and found a pretty good list of how it's viewed, and it even breaks down different Christian denominations, but I couldn't find anything about treatment for Islam from the link but we have a page here. Most of the others don't have full blown jurisprudence, so I figure the treatment probably involves some rites or counseling or something, or maybe expulsion, but the write-ups make that seem unlikely. - Lambajan 15:01, 6 September 2008 (UTC)[reply]
All I can say is that us atheists screw like bunnies, and we're loving it! 83.250.202.36 (talk) 22:46, 6 September 2008 (UTC)[reply]
On the Baha'i Faith, taken from the Kitab-i-Aqdas: God hath imposed a fine on every adulterer and adulteress, to be paid to the House of Justice: nine mithqals of gold, to be doubled if they should repeat the offence. The authorised notes state that this applies to unmarried individuals, not to married persons doing the deed with someone other than their spouse. It is not, however applied at the moment, and is reserved "for a future condition of society" (quoted from the notes to the standard edition). It's been emotional (talk) 15:31, 7 September 2008 (UTC)[reply]
Hmmm... I stand corrected. - Lambajan 01:45, 8 September 2008 (UTC)[reply]

Does Barack Obama speak a language other than english?

If he does, is he a fluent speaker? —Preceding unsigned comment added by 203.214.87.1 (talk) 12:07, 5 September 2008 (UTC)[reply]

When in Indonesia, he went to a school taught in the Indonesian language. That does not mean that he could understand the language. Even if he did learn it at that time, he doesn't necessarily know it now. Also, nearly all U.S. colleges require an introductory level of foreign language (usually French or Spanish). Students do not become fluent at that level. That is what was required of his education. I do not know what he learned beyond it. -- kainaw 12:12, 5 September 2008 (UTC)[reply]
This source has Obama saying that he is not fluent in languages other than English. However, this secondhand source indicates that, though not fluent, he has a 'smattering' of Spanish, Indonesian, and Swahili. -FisherQueen (talk · contribs) 12:14, 5 September 2008 (UTC).[reply]
Thank you both. I didn't expect such a quick response. 203.214.87.1 (talk) 12:23, 5 September 2008 (UTC)[reply]
I'm sure he is very fluent in the Political dialect of Doublespeak (as is John McCain). Clarityfiend (talk) 18:35, 5 September 2008 (UTC)[reply]
If a politician couldn't speak that, we'd never elect them. C'est la vie! Fribbler (talk) 18:43, 5 September 2008 (UTC)[reply]
Probably. But that proposition has never actually been tested, to my knowledge. :) -- JackofOz (talk) 21:24, 5 September 2008 (UTC)[reply]

Queen Victoria's funeral cortège: Edward and Wilhelm?

Caption for this image is: "Detail from the photograph above of Queen Victoria's Funeral Cortège. King Edward VII is clearly seen in the centre immediately behind the coffin, with the German Kaiser to his right."

I'm not up on my turn of the century royalty. It seems to be indicating that Edward is dead center of the image (in dark dress) with Wilhelm to the left of him (also in dark). Or are they the fellows behind those fellows (with "Wilhelm" dressed lighter tan the others)? --98.217.8.46 (talk) 14:12, 5 September 2008 (UTC)[reply]

Edward's the rather dumpy one with a dead bird on his hat, Kaiser Bill (rather skinny) is to his right (our left), with his left hand resting on his sword, and another dead bird on his titfer, and excessively large lapels. DuncanHill (talk) 14:16, 5 September 2008 (UTC)[reply]
Thanks, that helps a lot! --98.217.8.46 (talk) 14:35, 5 September 2008 (UTC)[reply]
Just to clarify (in case anyone is still confused), your first interpretation was the right one, both in dark dress, Edward's left hand is obscured by the head of the gun-carriage attendant, and you can make out the Kaiser's over-groomed moustache. DuncanHill (talk) 14:39, 5 September 2008 (UTC)[reply]
So in this photo
File:FuneralVictoriaDetail3.jpg
, the red arrow points to Edward, and the blue arrow to the Kaiser. Any idea who the slightly more dumpy guy is that the yellow arrow points to, or the exceedingly dumpy one shown by the green arrow? - Nunh-huh 15:03, 5 September 2008 (UTC)[reply]
Oscar_II_of_Sweden was probably there, and i just get the general feeling that he's the guy by the green arrow. It kind of looks like him, comparing to the picture in his article. Complete guess on my part though, so don't trust it in any way..../Coffeeshivers (talk) 17:47, 5 September 2008 (UTC)[reply]
Reading the linked old Times article it turns out he wasn't there, only his son, the future Gustav V. I don't think he ever had a particularly large beard, so green arrow probably isn't him /Coffeeshivers (talk) 09:32, 7 September 2008 (UTC)[reply]
(ec)Yes, yes, and looks vaguely Russian? I'd like to know who the frankly portly chap to our left of the yellow arrowed-fellow is too. DuncanHill (talk) 17:49, 5 September 2008 (UTC)[reply]
I don't know, but I think the protocol of 1901 would have some trouble in arranging Victoria's two surviving sons and an assortment of emperors and kings in the correct order in the wake of the coffin. Somewhere near the front will be the Duke of Connaught. Kaiser Bill, although only a grandson, seems to have got up to the front because he was also an emperor. Strawless (talk) 12:38, 6 September 2008 (UTC)[reply]
He was her eldest grandson, and had attended her at her deathbed. DuncanHill (talk) 12:41, 6 September 2008 (UTC)[reply]
Is it Connaught to Edward's left? DuncanHill (talk) 12:53, 6 September 2008 (UTC)[reply]
It's too blurred to tell from the picture, but he was entitled to the plumes, and it's hard to think who else would be there. Strawless (talk) 13:13, 6 September 2008 (UTC)[reply]
It's Connaught, per [7]. DuncanHill (talk) 13:20, 6 September 2008 (UTC)[reply]
For whom were plumes reserved? - Nunh-huh 13:50, 6 September 2008 (UTC)[reply]

Could a minor face life imprisonment if committs murder in the UK?

If a minor kills his parents (example). Could he face life in prison? Being minor... --190.49.99.40 (talk) 21:42, 5 September 2008 (UTC)[reply]

See At Her Majesty's Pleasure - "Where a person convicted of murder or any other offence the sentence for which is fixed by law as life imprisonment appears to the court to have been aged under 18 at the time the offence was committed, the court shall (notwithstanding anything in this or any other Act) sentence him to be detained during Her Majesty’s pleasure". This is not to be construed as legal advice, rather as an educated layman's comment. DuncanHill (talk) 21:45, 5 September 2008 (UTC)[reply]

September 6

Renaissance police?

What was the equivalent of city guards or police in Da Vinci era Italy or similar high-tech renaissance metropolises?--Sonjaaa (talk) 00:26, 6 September 2008 (UTC)[reply]

Police#History might help. It looks like there wasn't really anything resembling a modern police force until a couple of centuries after Da Vinci. --Tango (talk) 00:33, 6 September 2008 (UTC)[reply]
They may have had Watchmen, but there is no mention of Italy (or the rest of continental Europe) in that article. --Tango (talk) 00:39, 6 September 2008 (UTC)[reply]

Is there a gender-neutral term for a watchman? Is the force called the "city watch" or other?--Sonjaaa (talk) 01:15, 6 September 2008 (UTC)[reply]

At the time, they almost certainly would have been men. "City watch" would be an accurate description, I don't know if it was ever used as a name. --Tango (talk) 02:45, 6 September 2008 (UTC)[reply]
I don't think Italy had the PC police... --mboverload@ 03:30, 6 September 2008 (UTC)[reply]
I don't think Italy has the PC police. :-) Fribbler (talk) 14:44, 6 September 2008 (UTC)[reply]

Britian and soldiers

Why do I keep reading stories about soldiers in Britain being disrespected/turned away for services? This is a compilation of various news articles [8]. More info [9]. Basically, a solider was turned away from a hotel in Britain because he was a solider. I believe that most people in America would gladly give their rooms to any solider in need, but airlines and hotels turn them away in Britain. If this ever happened in America (other than the Airlines charging soldiers for extra bags) there would be hell to pay. Why is this? Is this because Britain never had a Vietnam, where the public blamed the soldiers instead of the war, something the US has since learned from? I realize this doesn't represent Britain as a whole, but I keep hearing these stories...--mboverload@ 03:28, 6 September 2008 (UTC)[reply]

in recent months the hotel had “experienced some rather serious incidents” resulting from personnel from a local barracks staying at the hotel, and said staff had been requested to act with caution when taking future bookings from members of the armed forces.. Corvus cornixtalk 04:16, 6 September 2008 (UTC)[reply]
Well done Corvus! Can I ask what you did exactly to find it? Did I just skim over that little factoid? --mboverload@ 04:19, 6 September 2008 (UTC)[reply]
I looked up tomos on google news and found this. Corvus cornixtalk 04:21, 6 September 2008 (UTC)[reply]
Ah, interesting how they put that mitigating factor at the END of the long article. Thank you much Corvus. --mboverload@ 04:24, 6 September 2008 (UTC)[reply]
For it's Tommy this, an' Tommy that, an' "Chuck him out, the brute!", But it's "Saviour of 'is country" when the guns begin to shoot..., not a new phenomenon. [10] DuncanHill (talk) 09:54, 6 September 2008 (UTC)[reply]
In the BBC's People's War project, the Rev. Paul Wilkinson recalled VJ Day: "We went to Brecon, a big barracks town. I remember with some amusement seeing lines of soldiers all lying on the pavement all paralytically drunk!" Strawless (talk) 12:23, 6 September 2008 (UTC)[reply]
Whether it is deserved or not, "squaddies" generally do not have a good reputation in the UK (a google search of "squaddie reputation" will yield you a fine selection of opinion). It is quite conceivable that a hotel in a town like Woking (within pub-crawl distance of Aldershot) has a blanket ban on soldiers: a case of a rowdy minority make life difficult for the rest, perhaps. Gwinva (talk) 23:17, 8 September 2008 (UTC)[reply]

Nonstop US war

How long has the US been in constant war with different countries all together? I have heard 30 years and ive heard 50 years, not sure whats right. Any link would be appreciated. Thanks —Preceding unsigned comment added by 75.82.33.31 (talk) 06:42, 6 September 2008 (UTC)[reply]

I don't think they're in an official state of war with anyone at the moment. Do you mean just being involved in some kind of conflict? --Tango (talk) 06:47, 6 September 2008 (UTC)[reply]
Yes, arent we in a kind of war in Iraq? —Preceding unsigned comment added by 75.82.33.31 (talk) 06:57, 6 September 2008 (UTC)[reply]
No, the Iraqi government is friendly to the US, the US troops there are dealing with insurgents. --Tango (talk) 07:13, 6 September 2008 (UTC)[reply]
That's because the Iraqi government was put in place by the US. When the US first invaded, the Iraqi government of course resisted. It really doesn't make sense to say they are friendly to the US, when the US established them in the first place. ScienceApe (talk) 21:47, 8 September 2008 (UTC)[reply]
It's called the Iraq War for a reason. — Twas Now ( talkcontribse-mail ) 10:09, 6 September 2008 (UTC)[reply]
What it's called and what it officially is are two different things. If you're looking for the official, war-has-been-declared sort of war, we are not currently in any wars, but we were from like 2002 to 2007. Before that, we weren't in any official wars since... what, the Persian Gulf War? If you're talking unofficially we-have-soldiers-in-other-countries-fighting-a-lot, sure, we've probably had that sort of thing since around 2002. Not constantly for 30 or 50 years, though. --Alinnisawest,Dalek Empress (extermination requests here) 13:20, 6 September 2008 (UTC)[reply]
Have a look at War in Afghanistan (2001–present) if you think you're not currently involved in any official wars. Malcolm XIV (talk) 16:13, 6 September 2008 (UTC)[reply]
I would say "Iraq War" only refers to when the coalition was fighting Saddam's forces, we won that war, now it's just combating insurgency. --Tango (talk) 20:52, 6 September 2008 (UTC)[reply]
You're against common usage on that one, I think. Algebraist 20:55, 6 September 2008 (UTC)[reply]
Whether or not the war has been declared is irrelevant. If it looks like a duck, swims like a duck, and quacks like a duck, then it probably is a duck. — Twas Now ( talkcontribse-mail ) 23:57, 6 September 2008 (UTC)[reply]
The US was involved in military conflicts pretty much throughout the entire length of Bill Clinton's term, AFAIK (Kosovo, Somalia, etc.). So if we weren't in any military conflicts between 2000-2002 (which I also find doubtful), it would have been a small respite. The Jade Knight (talk) 09:45, 8 September 2008 (UTC)[reply]
The US last declared war in WWII. Everything since then has been an exercise of presidential power, usually with the consent of Congress. See Declaration of war by the United States and List of United States military history events. Whether we've been in a state of war for 30 years depends on how you choose to define the word "war". Consider the 1990s in Iraq and our relationship with North Korea for the last 50 years for examples of grey areas. --Sean 17:21, 6 September 2008 (UTC)[reply]
I think the OP is asking the total duration of all of America's wars. If so, United States casualties of war lists the wars and the years in which they were fought. By my count (rounding up to the nearest year), I get 9 for the Revolutionary War, 4 for the War of 1812, 3 for the Mexican-American War, 5 for the not-so-Civil War, 1 for the Spanish-American, 2 for WWI, 5 for WWII, 4 for the Korean War (although you might consider this just a "police action" or alternately much more, since there was never a formal treaty ending it), who knows how many for the Vietnam War, 2 for the Gulf War, and ? for the Iraq War. Even without counting the hard-to-determine Vietnam and Iraq Wars and various minor conflicts, that's 31 years already. So 30-50 is a reasonable guess. Clarityfiend (talk) 19:26, 6 September 2008 (UTC)[reply]
The Civil War was 4 years - April of 61 to April of 65. Corvus cornixtalk 21:32, 6 September 2008 (UTC)[reply]
I did say rounding up. Clarityfiend (talk) 00:10, 7 September 2008 (UTC)[reply]
If you add in the Indian Wars, the numbers go way up -- add about a century to your calculation. --Carnildo (talk) 23:52, 8 September 2008 (UTC)[reply]
As noted astutely above, what really constitutes "war" is a matter interpretation and of much debate. Gore Vidal (who, it must be said, isn't the Avatar of NPOV) writes "since VJ Day we have been engaged in what historian Charles A. Beard called 'perpetual war for perpetual peace'" in his book of that name, and gives a table (sourced from Federation of American Scientists) listing all the US "wars" since that time; this list seems to cover that entire era. FAS takes a particularly broad definition of "war" in this list, even including stuff like Operation Garden Plot and Operation Steel Box. -- Finlay McWalter | Talk 19:45, 6 September 2008 (UTC)[reply]

Let's not forget the Cold War. DOR (HK) (talk) 01:20, 8 September 2008 (UTC)[reply]

Czech Population in Hong Kong

I am wondering exactly how many Czechs live in Hong Kong. Vltava 68 (talk, contribs) 07:00, 6 September 2008 (UTC)[reply]

Try this site on the Hong Kong census of 2001. Strawless (talk) 12:12, 6 September 2008 (UTC)[reply]
The census and statistics only mention the very vague term whites; there is no detailed breakdown. According to a document of the Czech Foreign Ministry, there are 240 Czechs in China, including Hong Kong, but I only want the number in Hong Kong. Vltava 68 (talk, contribs) 20:37, 6 September 2008 (UTC)[reply]

The Hong Kong Government doesn't keep statistics such as you seek. Hence, the only reliable source would be the Czech Consulate or Embassy. DOR (HK) (talk) 01:21, 8 September 2008 (UTC)[reply]

...Which I just called to get the information from (around 110-120 Czechs). Incidentally, does the HK American consulate have a website? Vltava 68 (talk, contribs) 09:28, 8 September 2008 (UTC)[reply]
hongkong.usconsulate.gov/webmaster.html DOR (HK) (talk) 01:20, 9 September 2008 (UTC)[reply]

Nathaniel Woodard painting

Hi! Due to problems with German copyright laws I am looking for the painter of this picture. Who can help?

--Operarius (talk) 09:21, 6 September 2008 (UTC)[reply]

(the date of publication might help as well!) --Operarius (talk) 09:35, 6 September 2008 (UTC)[reply]
Could that be the portrait of him by C. G. Anderson at Lancing College? If it is, I expect they could tell you the date. Strawless (talk) 12:02, 6 September 2008 (UTC)[reply]

Has anyone read the book 'Tales from Ovid' by Ted Hughes?

Please describe what you thought about it. Did you like it? Any specific reactions? —Preceding unsigned comment added by 81.151.147.129 (talk) 12:08, 6 September 2008 (UTC)[reply]

Um, *cough*, if you're looking for things like a reaction to the book for an English class, sorry, we don't do homework here. --Alinnisawest,Dalek Empress (extermination requests here) 13:16, 6 September 2008 (UTC)[reply]
If I had read it, I'd never tell tales. --- OtherDave (talk) 13:40, 6 September 2008 (UTC)[reply]
If you're want book reviews, this isn't the place. There are plenty of book reviews on the internet, try google. --Tango (talk) 20:49, 6 September 2008 (UTC)[reply]
I've read Metamorphoses, but I don't suppose that qualifies. 83.250.202.36 (talk) 22:41, 6 September 2008 (UTC)[reply]

china

This question has been removed. Per the reference desk guidelines, the reference desk is not an appropriate place to request medical, legal or other professional advice, including any kind of medical diagnosis, prognosis, or treatment recommendations. For such advice, please see a qualified professional. If you don't believe this is such a request, please explain what you meant to ask, either here or on the Reference Desk's talk page.
This question has been removed. Per the reference desk guidelines, the reference desk is not an appropriate place to request medical, legal or other professional advice, including any kind of medical diagnosis or prognosis, or treatment recommendations. For such advice, please see a qualified professional. If you don't believe this is such a request, please explain what you meant to ask, either here or on the Reference Desk's talk page. --~~~~

Politician’s position on the issues

I stumbled across a very informative site, which I have been looking for, for a very long time. It lists the political positions of every candidate on every issue. The site is at On The Issues.

The coverage is so detailed in fact that I needed to create a decision table to reveal conditions, combinations and positions which have not been addressed.

My question is whether or not any political position decision tables exist elsewhere on the Internet or not?

Example of political position decision table:
Position of Sarah Palin as of Sept. 6, 2008
Condition Combination
pregnancy threat to mother's life Yes No Yes No
baby will have Down's Syndrome Yes Yes No No
Action Position
Abort pregnancy Yes No Yes No


This question was removed for being spam and then reinstated when the question was discovered. -- kainaw 21:51, 6 September 2008 (UTC)[reply]
I doubt positions of that level of (unnecessary) detail exist. (You could sum up Palin's view in a sentence, why use 12 cells to convey the same information? "She believes in abortion only when continuing the pregnancy would be a threat to the mother's life" is a lot clearer than your table). Such a table would obscure nuance more than it would enlighten it, IMO. --98.217.8.46 (talk) 22:02, 6 September 2008 (UTC)[reply]
Its not about Palin... its about seeing which cells are not yet complete. If you visit the site On The Issues you will see that in the case of any politician there are a mired of issues on which they hold positions - far too many to get into your own head for analysis at one time, unless you are the politician. —Preceding unsigned comment added by 71.100.2.227 (talk) 22:26, 6 September 2008 (UTC)[reply]
Well then maybe we can include a link to the politician's page at that website in the "External links" section. — Twas Now ( talkcontribse-mail ) 23:49, 6 September 2008 (UTC)[reply]

The most important authentic relic?

You hear all of the time about these relics that are said to belong to Jesus or some other Christian muckety-muck, but when you examine them, it turns out that it really is just a black linen duvet or something. But some of them surely must be authentic (authentic in the sense that they really are what they're presented as, not authentic as in "can do magic"). I mean, it wouldn't be so hard just to go to the grave-site of St. Oswald-who-ate-to-many-poppies and just pick up his jawbone or whatever. So my question is this: what's the most valuable or most prized relic where scholars are somewhat certain of its authenticity (I'm just talking Christian relics now, if we allowed for any religion, I would suppose the Black Stone in the Kaaba in Mecca would probably win). 83.250.202.36 (talk) 22:38, 6 September 2008 (UTC)[reply]

Well, the most popular is probably the Shroud of Turin. —Preceding unsigned comment added by 71.100.2.227 (talk) 22:51, 6 September 2008 (UTC)[reply]
The shroud isn't genuine. Algebraist 22:56, 6 September 2008 (UTC)[reply]
(my bad little "turns out to be a black linen duvet"-joke was actually a reference to the Shroud of Turin :) 83.250.202.36 (talk) 23:00, 6 September 2008 (UTC)[reply]
Why do you say that? —Preceding unsigned comment added by 71.100.2.227 (talk) 23:11, 6 September 2008 (UTC)[reply]
Because there is absolutely no evidence to suggest it is genuine and plenty of evidence to suggest it isn't. Sure, people have picked holes in every test which has shown it to be a fake, but they haven't done any conclusive tests to show otherwise (at least, not that I know of). --Tango (talk) 00:17, 7 September 2008 (UTC)[reply]
One test I find interesting by the Jet Propulsion Laboratory is the correlation between the intensity of reflection or (degree of transparency) and distance from the surface. I know that similar effects can be achieved with vapor disposition on or alteration of a surface either from degree of concentration with same distance or from a difference in distance with the same concentration in absolutely still air. This fact does more to support the possibility of genuineness than to refute it, as do many of the other tests, sufficiently enough to mark anyone rejecting the possibility of genuineness as a sophomoric. — unsigned comment added by 71.100.2.227 (talk · contribs)
That has absolutely no bearing on whether or not it is the burial shroud of Jesus. — Twas Now ( talkcontribse-mail ) 05:55, 7 September 2008 (UTC)[reply]
I've always thought that the fact that shroud-man looks like a frickin' Viking makes the whole thing kinda preposterous. But that's just me. 83.250.202.36 (talk) 07:40, 7 September 2008 (UTC)[reply]
I find, in fact, quite the opposite in Pontius Pilate's reaction to Jesus Christ as a man, in finding no flaw in him. Surely if Jesus looked like a dark skinned crooked nose Jew Pilate might not have taken so much time to question and examine him or otherwise been so generous, nor the Roman soldiers far less so, seeing him as an actual physical threat. —Preceding unsigned comment added by 71.100.2.227 (talk) 12:18, 7 September 2008 (UTC)[reply]
Are you saying that Jesus was not a Jew? That would be quite some statement. As for the colour of his skin or the shape of his nose, or any other of his physical characteristics, we simply don't have any way of knowing, and the Shroud of Turin certainly doesn't count as evidence for his appearance. Neither do the traditional depictions of him. -- JackofOz (talk) 14:19, 7 September 2008 (UTC)[reply]
St Stephen I's right hand? DuncanHill (talk) 22:53, 6 September 2008 (UTC)[reply]
Very interesting! Thanks! Any other? 83.250.202.36 (talk) 07:40, 7 September 2008 (UTC)[reply]
Although not directly answering your question, 83.250.202.36, you may enjoy reading about the Titulus Crucis (and a link to Thiede's book at Amazon.com).
Regards, Ev (talk) 23:28, 6 September 2008 (UTC)[reply]
Definitely not authentic, but my favorite is probably Jesus' foreskin. — Twas Now ( talkcontribse-mail ) 14:26, 7 September 2008 (UTC)[reply]
Not to mention the Holy Umbilical Cord. Corvus cornixtalk 20:56, 7 September 2008 (UTC)[reply]

Relics as grave robbery

The above question has me thinking about relics. I think most major religions respect the dignity of the dead, and to dig up someone's remains is considered a despicable act. I would expect that to desecrate the grave of a saint/martyr might be considered even worse (to the religious authorities) than to do so to a random person's grave. Yet religious bodies have relics in their possession, so implicitly, they condone grave robbery.

I understand that most relics are (supposedly) hundreds of years old, so religious bodies presently don't have newly acquired relics that need to be "justified". That leaves me with these questions:

  1. In the past, how did religious authorities, such as the Catholic Church, justify the acquisition of relics that are pieces of people's bodies (or other items removed from their grave)?
  2. How do current religious authorities justify the keeping of relics that are pieces of people's bodies (or other items removed from their grave)?
    • I assume money (i.e. revenue from tourist/pilgrim donations) has something to do with it.
  3. If I donated a saint/martyr's body part, which the authorities believed to be authentic, to a religious body, what actions would/should the authorities take?
    • e.g. accept it / deny it / investigate me for criminal charges / …

I am mainly interested in the views (past and present) of Catholic / Orthodox Christian churches, but would love to hear the views from other religions, too! — Twas Now ( talkcontribse-mail ) 23:44, 6 September 2008 (UTC)[reply]

Relic#Christian_relics makes reference to an account in 2 Kings in which the bones of Elisha bring a dead man to life. In addition, supposedly the early Christians in Rome prayed among tombs in the catacombs, leading to the practice of placing a saint's relics in the altar stone. This was not seen as grave robbery, but as respect for / veneration of the saintly individual.
Here's an article from the Boston Globe. It mentions the problem of dealing with altar stones from churches that have closed.
Without pretending to be an authority, I'd say that the most common Catholic justification for retaining relics, rather than burying them, is to allow the faithful to make a spiritual connection with the saint and through him or her, with God. Certainly tourism plays some part, though I think that was far more common in the middle ages ("the holy blissful martyr for to seek...") than today.
Authorities would question the provenance (so to speak) of any body that you claimed was that of a saint; most likely, the authorities would feel they'd had a good handle on these things.
By the way, the Catholic church has classes of relics. You're speaking of first-class relics, I think: parts of the person's body like hair or bone. (The bodies of some saints have been preserved 'uncorrupted.') See relics for more. --- OtherDave (talk) 20:22, 7 September 2008 (UTC)[reply]

Technicolor effect

Is it possible to replicate the technicolor "effect" using say, video recorded via iphone? If so, what programs would I look into? What processes? Etc...

Kenjibeast (talk) 23:37, 6 September 2008 (UTC)[reply]

I think you'd get better responses asking this at the Computer desk rather than here at Humanities. Dismas|(talk) 00:16, 7 September 2008 (UTC)[reply]

September 7

Is United States of America Bankrupt?

Are the vast majority of people in USA getting poorer (financially) by the year? 122.107.177.150 (talk) 01:48, 7 September 2008 (UTC)[reply]

Well, the unemployment rate just reached 6.1%, the gap between rich and poor has bee steadily widening, the cost of just about everything went up quite a bit recently while wages largely stayed the same (but many businesses are losing employees by attrition and replacing them when they can by lower paid employees, so wages are effectively going down). This is all over the news, I don't see the need to ask the reference desk. If you want opinions about why... well, there's facts and there's political posturing, both of which are available from many sources, wikipedia included. - Lambajan 03:16, 7 September 2008 (UTC)[reply]
Though, it must be said that minimum wage has increased drastically in the US over the last couple of years. So the least paid people in the US make more now than they did 5 years ago. The Jade Knight (talk) 09:51, 8 September 2008 (UTC)[reply]
If you mean over the long term, I don't expect so. I've not seen the figures for the US but the UK figures show real terms growth across the board. The rich are getting richer faster than the poor, but the poor are still getting richer. I would expect the US figures to be similar. --Tango (talk) 03:37, 7 September 2008 (UTC)[reply]
Are the poor becoming richer in real terms or nominal terms? — Twas Now ( talkcontribse-mail ) 03:40, 7 September 2008 (UTC)[reply]
As I said, it's real terms growth (assuming I'm remembering correctly - I'll try and find the numbers). --Tango (talk) 04:03, 7 September 2008 (UTC)[reply]
Either I'm remembering incorrectly, or the US and UK aren't similar in that respect. I just found this saying median income in the US dropped 1.1% in real terms between 2000 and 2006 (it apparently grew since then, though). I'll keep trying to find the UK figures that I'm remembering. --Tango (talk) 04:09, 7 September 2008 (UTC)[reply]
It's a common misconception that (in the US) the poor are becoming poorer. In fact, the poor are becoming richer (in real terms). The reason people quote a rise in the income gap is because the rich are becoming richer (in real terms) *faster* than the poor are becoming richer (in real terms). Wikiant (talk) 19:18, 7 September 2008 (UTC)[reply]
The article I link to above would cast that into doubt. --Tango (talk) 02:19, 8 September 2008 (UTC)[reply]
Your article is in error. Check the Bureau of Labor Statistics publication, "Statistical Abstract of the United States" 2006 edition, Table 673. A summary is here.

The last time nominal GDP per capita declined was 1958. Other years of decline since data was first collected (1929) are 1930, 1931, 1932, 1933, 1938, 1946, 1949 and 1954. For nominal disposable personal income, deduct 1946, 1949, 1954 and 1958 (i.e., the last decline was in 1938).

In real (Chain-linked) terms GDP per capita declined 0.3% in 2001, and real disposable personal income declined last in 1993.

Both measures declined in 1930, 1931, 1932, 1933, 1938, 1945, 1946, 1947, 1949, 1954, 1958, 1974 and 1991. Real GDP per capita alone declined in 1970, 1975, 1980 and 1982. Source: http://www.bea.gov/bea/dn/nipaweb/TableView.asp?SelectedTable=253&ViewSeries=NO&Java=no&Request3Place=N&3Place=N&FromView=YES&Freq=Year&FirstYear=1929&LastYear=2008&3Place=N&Update=Update&JavaBox=no DOR (HK) (talk) 01:41, 8 September 2008 (UTC)[reply]

That's interesting, but it might be more useful averaged out over each decade, or something. --Tango (talk) 02:19, 8 September 2008 (UTC)[reply]


US GDP per Capita and Disposable Income per Capita:
Decade Nom. GDP pc Nom Dis Inc pc Real GDP pc Real Dis Inc pc
1930s -1.9% -2.2% +0.2% +0.3%
1940s +9.8% +8.9% +4.2% +2.7%
1950s +4.8% +4.5% +2.3% +1.9%
1960s +5.4% +5.3% +3.0% +3.0%
1970s +8.9% +9.1% +2.2% +2.4%
1980s +6.9% +7.4% +2.1% +2.3%
1990s +4.1% +4.0% +1.9% +2.0%
2000s +4.1% +4.1% +1.5% +1.5%


DOR (HK) (talk) 01:36, 9 September 2008 (UTC)[reply]

Need Chinese History Expert

A long time ago a read a book about a Chinese revolutionary, but I can't remember his name. Everything I remember about him:

-It was a man
-He had a "golden tongue" (meaning, he was "eloquent")
-When taken prisoner, he used his persuasive powers to convince his (communist?) captor to let him go. The captor, who was a soldier, hung himself, moved to tearful suicide by the revolutionary's speech.
-The revolutionary was captured many more times, but always convinced his guards to let him go, so amazing were his oratorical skills. 128.239.177.28 (talk) 01:42, 7 September 2008 (UTC)JustCurious[reply]

Sorry, not enough to go on. DOR (HK) (talk) 01:43, 8 September 2008 (UTC)[reply]

Palin and polar bears

Is the following really true and accurate? (ie: that S. Palin is suing the Bush administration): "The 44-year-old governor says a federal government decision to protect the polar bear will cripple energy development offshore. As a result, she is suing the Bush administration, which ruled the polar bear is endangered and needs protection."[[11]]. If it is, is she likely to win? --AlexSuricata (talk) 01:53, 7 September 2008 (UTC)[reply]

I'm rather doubtful that someone could sue an administration. She could conceivably sue the federal government, but it is highly unlikely (to the point of ridiculousness), seeing as this is an election year. This isn't a statement as to whether she would or would not do such a thing, of course, just saying that it's extremely unlikely. --Alinnisawest,Dalek Empress (extermination requests here) 02:25, 7 September 2008 (UTC)[reply]
Actually, this particular year, suing the current administration could be pretty shrewd politically. They're probably not bragging about it because the undecided independent voters that would appreciate a candidate who sued the current administration would probably be the same ones who would be turned away by a candidate who protects oil companies over polar bears. - Lambajan 14:35, 7 September 2008 (UTC)[reply]
A Google search [12] finds plenty of sources saying she sued the Bush administration but I haven't examined the legal technicalities. PrimeHunter (talk) 02:51, 7 September 2008 (UTC)[reply]

Gov. Palin could have authorized the Attorney-General of Alaska to sue the federal government challenging the designation of polar bears as a protected species. It would turn out to be politically embarassing now. It might be taken as a sign of independence from the Bush administration. Personally, I find her nomination an embarassment. Actually, it makes a lot of sense to me. I'm not commenting on the plight of artic bears, only whether such a suit is credible. I assume there are netural websites dedicated to finding the veracity of all the rumors.75Janice —Preceding unsigned comment added by 75Janice (talkcontribs) 02:55, 7 September 2008 (UTC)[reply]

Yes, she sued to have polar bears de-listed from the Endangered Species Act list because she was concerned that the categorization would hurt the economy of her state. She sued "the Bush administration" in the sense that she had her state sue the Department of the Interior, which is in charge of such listings. Bush himself likely has little to do with such things—those sorts of decisions are delegated pretty far down the line. --98.217.8.46 (talk) 13:30, 7 September 2008 (UTC)[reply]

ABOUT OLD FOLKS

WHAT IS THE SUITABLE ENTERTAINMENTS AND GAMES FOR THE OLD FOLKS AT THE OLD FOLKS HOME?118.100.195.105 (talk) 04:37, 7 September 2008 (UTC)[reply]

My brother-in-law recently brought great joy to the residents of his grandmother's nursing home by bringing in a portable karaoke machine and running it for them for a few hours. Crypticfirefly (talk) 05:12, 7 September 2008 (UTC)[reply]
I think old folks are generally as individualistic in their interests as young folks and folks in general. — Twas Now ( talkcontribse-mail ) 09:16, 7 September 2008 (UTC)[reply]
True, but there are trends. I've heard the wii is becoming quite popular with that lot. - Lambajan 14:37, 7 September 2008 (UTC)[reply]
Sexuality in older age? Clarityfiend (talk) 17:59, 7 September 2008 (UTC)[reply]
Pet therapy is common in nursing homes here in Australia. Steewi (talk) 04:54, 8 September 2008 (UTC)[reply]

People famous for hand-to-hand combat

The famous generals are well known, but in the Greek/Roman/Viking periods, or any other time before the invention of guns, were there people who are remembered for their skill in individual combat? Are there any historical figures who measure up to legendary people like Ajax of Telamon, Beowulf, or Bodvarr Bjarki? (I can think of one or two from American history, like Jim Bowie, but none earlier). 128.194.34.211 (talk) 05:27, 7 September 2008 (UTC)[reply]

Achilles, assuming he is based on a historical warrior. — Twas Now ( talkcontribse-mail ) 05:46, 7 September 2008 (UTC)[reply]
I'd put Cúchulainn up against Achilles any day (assuming that he was at some point a real guy, that is :). As for a real person who was bad-ass fighter, how about Leonidas I? Honestly I don't really think there are that many that are famous for it, like in the myths, for very practical reasons: you could have supernatural powers of close-combat fighting, but in an army of thousands it wont matter much. Especially when you're fighting in a phalanx (or similar), then everyone has to cooperate and use very specific techniques. If you improvise, showing off you're mad Jackie Chan-moves, then you'd just be breaking the line and degrade the performance of the company as a whole. The people who are remembered are the great strategists, the great generals. One of my favourite people from the Roman empire (well, Byzantine empire, but I've always thought that was a ridiculous distinction) is Flavius Belisarius, quite possibly the greatest military commander in the history of warfare (and the last man ever to receive a triumph). He could pull off miracles at the battlefield, no matter how many Achilleses the other side had. 83.250.202.36 (talk) 07:18, 7 September 2008 (UTC)[reply]

zh:李书文 was a master of bājíquán who never lost a duel, I hear. He died through poisoning. --Kjoonlee 08:32, 7 September 2008 (UTC)[reply]

Dioxippus. —Preceding unsigned comment added by 194.171.56.13 (talk) 10:23, 7 September 2008 (UTC)[reply]
Miyamoto Musashi. Algebraist 10:24, 7 September 2008 (UTC)[reply]


Also see the article Nine Worthies.--64.228.91.86 (talk) 21:58, 7 September 2008 (UTC)[reply]

More recently, Xu Shiyou. DOR (HK) (talk) 01:47, 8 September 2008 (UTC)[reply]

I'm with DOR: we don't need to go so far back. Let's add Wong Fei Hung and Bruce Lee to the list. --Shaggorama (talk) 08:57, 9 September 2008 (UTC)[reply]
Marcus Sergius --Tagishsimon (talk) 09:01, 9 September 2008 (UTC)[reply]
Still is the story told, How well Horatius kept the bridge, In the brave days of old. DuncanHill (talk) 11:38, 9 September 2008 (UTC)[reply]

Plight of Charter 77 Signatories

I am wondering what happened to the Charter 77 signatories, as the article is rather vague. On of the links there had all the names of the signatories, but when I searched for the names, the results were all in Czech, and I very much doubt that the list includes updates (i.e. marriage, which would probably result in a change of family name)[13]. Could someone tell me what happened to the lesser well known signatories (i.e. I'm rather intrigued (by the print handwriting, which reminds me a bit of the one I used to use!) about the fate of a Marie Švermová)? Vltava 68 (talk, contribs) 08:52, 7 September 2008 (UTC)[reply]

The German WP has a short list of signatories, some of which (Vaclav Havel, Pavel Kohout) have articles in the English WP. You can check those via the language links. The Czech version has a much longer list of signatories (again, you can check any English parallel articles via the language links), but neither of the lists include Marie Svermova. There is an article on such a person (1902 - 1992) in the Czech WP, but it does not seem to mention Charta 77. I assume her to be the Marie Svermova (widow of Jan Sverma) who was arrested in the purge of the Communist Party in 1950 and was released in 1956. She is briefly mentioned as the host and guide of young Bill Clinton on his visit to Prague in the 1960s, but I can´t find anything on her activities beyond that. --Cookatoo.ergo.ZooM (talk) 10:31, 7 September 2008 (UTC)[reply]

Racism or learning from experience?

I found the following sentence on an HR web-site:

"Researchers at a university in Germany - where the right to be surly is almost a national pastime - now claim enforced jolliness on the job is much more likely to make people fall ill."

and I ask myself if it is racism or just experience (that can be right or wrong). Is any form of negative comment against group X a form of discrimination? —Preceding unsigned comment added by 88.6.158.246 (talk) 12:35, 7 September 2008 (UTC)[reply]

Moved from Science desk
Are you saying the negative comment is that in "Germany [...] the right to be surly is almost a national pastime"? I don't see it as a negative comment, but it is a stereotype (see Stereotype#German stereotypes). — Twas Now ( talkcontribse-mail ) 12:49, 7 September 2008 (UTC)[reply]
I've never heard a Germans-are-surly stereotype; the main ones that I'm familiar with are the two somewhat contradictory ones of Germans as jolly lederhosen-and-dirndl-clad Oktoberfest beer drinkers, and Germans as soulless petty bureaucrats who don't care whether what they do is right or wrong, as long as the proper forms are correctly filled out in triplicate. The arrogant militaristic Prussian aristocrat with a duelling scar stereotype is quite old fashioned by now (though it did influence Fearless Leader), but one rising steroetype among British people seems to be that at resorts Germans try to ensure that only other Germans use beach-chairs by assigning someone to sneak down early each morning and put towels on the available beach-chairs en masse, to reserve them (there have been several news stories about spontaneous British beachchair-towel revolts at swimming places frequented by both British and German tourists...). AnonMoos (talk) 17:35, 7 September 2008 (UTC)[reply]
Well, in my experience German people are not notably surly. In answer to your question, I would say that any broad negative generalization about a group could be considered to be a sign of prejudice. Whether it amounts to discrimination would of course depend on the effect such a comment had, and on the power-relationship between the person making the comment, and members of the group so characterized. DuncanHill (talk) 12:47, 7 September 2008 (UTC)[reply]
I disagree. Observing something that applies on average to a group of people (whether good or ill) is not prejudice. For example, it is a fact that (in the US) the average black person is more likely to be incarcerated than the average white person. Neither is it prejudice to assume that *given no other relevant information* a given black person is more likely to have been incarcerated than a given white person. It becomes prejudice when other relevant information is ignored in favor of the single piece of information of the person's skin color. In short, the distinction here is one between making a statistical observation (not prejudice) and inappropriately weighting the statistical observation (prejudice). Let me anticipate the counter-argument that, "it is because society is racist that blacks are more likely to be incarcerated than whites." That's a fair hypothesis, but not germane to my argument -- I'm *not* saying that blacks are more prone to be criminals; I'm saying that (statistically) blacks are more likely to be incarcerated. Wikiant (talk) 13:23, 7 September 2008 (UTC)[reply]
There is a difference between "could be considered to be" and "is". DuncanHill (talk) 13:24, 7 September 2008 (UTC)[reply]

I think in this particular case it's not really saying anything. "The right to" is their back door out of trouble. Most people would read this as that interjection is just there to set up the inherent humor of the finding, but if they would take offense to being called surly then they would figure by the way it's written that they're not being called surly but that they respect the freedom to be that way if they choose. The fact that this is really rather harmless helps their weaseling. If they were to talk about a poor communities' right to be jobless and say it's a pastime it would be a much different story. I'm part Irish and the Notre Dame mascot doesn't bother me nearly as much as Chief Wahoo simply because the days of real Irish persecution in my country are long over while racism and prejudice towards First Nations Peoples persists. While that may seem like an aside, I think it gets at the heart of the issue. You can make any sort of outlandish jab at your friends and they take it in stride because they know they're ok with you, but if things actually aren't ok then you have a problem and you need to be careful what you say. - Lambajan 15:09, 7 September 2008 (UTC)[reply]

Just in case anyone is interested, this [14] is the source of the quotation. DuncanHill (talk) 15:13, 7 September 2008 (UTC)[reply]

Pink Houses at the DNC?

I seem to recall Pink Houses, or a snippet of it, being played at some point at the 2008 Democratic National Convention. I was just wondering if I am remembering correctly. (I know that Mellencamp told McCain to stop using his songs, and let Edwards use them on his campaign, but I'm specifically wondering about whether they played it at the DNC.) --98.217.8.46 (talk) 15:10, 7 September 2008 (UTC)[reply]

Looking at the projected tracking map for Hurricane Ike (here [15] or here [16]) that shows it bouncing up the full length of Cuba, I wondered what, if any, precautions will be taken at Guantanamo Bay in the event that such a storms threatens its security. I couldn't find anything on line. Does anyone have a link even to a discussion of it? ៛ Bielle (talk) 15:49, 7 September 2008 (UTC)[reply]

I found an answer to my own question:
Strong gusts and steady rains fell at the U.S. naval base at Guantanamo Bay in southeast Cuba, where all ferries were secured and beaches were off limits. The military said cells containing the detainees — about 255 men suspected of links to the Taliban and al-Qaida — are hurricane-proof. But the base was spared the strongest winds. [17]
I don’t know if it is the correct answer, though. ៛ Bielle (talk) 00:55, 8 September 2008 (UTC)[reply]
"Hurricane proof???" Edison (talk) 03:25, 8 September 2008 (UTC)[reply]
Which is probably another way to say that it was built to withstand a full-on military attack of some sort. --98.217.8.46 (talk) 13:14, 8 September 2008 (UTC)[reply]

'Securitised mortgages'

Hello wikipedia,

One of the causes of the credit crunch is, apparently, that whilst banks used to limit the amount they borrowed to the amount of money people had saved with them, now banks can 'securitise' mortgages so they can lend much more. So i guess my questions are these, firstly, what is a securitised mortagage and secondly, if i wanted to buy a securitised mortgage (if i was an investor), why would i? (i'm told that when banks lend to people who will basically default, they securitise the mortgage, but why would any one want to buy this -what are you actually buying?) Thanks,82.22.4.63 (talk) 16:25, 7 September 2008 (UTC)[reply]

We have an article on Securitization, but I am not sure how helpful it will be. DuncanHill (talk) 16:29, 7 September 2008 (UTC)[reply]
First, the opening statement to the question is debatable. Credit is often tight but I have never heard of a limit on borrowing equal to the amount the borrower already has on deposit. I know that talking to bankers always makes me feel as if that is what they want, but I don't think it is true. As to mortgages, they are, by definition, secured borrowing. The real property is itself the security for the loan whether the money borrowed is used to purchase the property or for other purposes as permitted by the lender. I am not sure how one might otherwise "securitise" a mortgage. ៛ Bielle (talk) 16:47, 7 September 2008 (UTC)[reply]
That's not what securitized means; it means "turned into a financial product", not "made more secure". 87.114.2.167 (talk) 16:49, 7 September 2008 (UTC)[reply]
"securitized" means that a big bunch of different mortgages have been grouped together into one big pool, and that pool is chopped up and sold as a security. There are lots of other kinds of securities - they're all backed by some kind of collateral or another (bits of the ownership of companies, a share of government debt, etc.). The handy thing about a security is that it's generic - you don't need to know much about what's inside it to buy and sell it. As property prices were going up, lots of financial companies thought they'd like to invest in this market, but they didn't know much about property and didn't want to bother with the details of hundreds or thousands of individual mortgages in lots of different places. So other companies securitized their mortgages (making mortgage-backed securities), and ratings agencies gave those securities scores. Based on those scores large companies could buy and sell mortgage value without knowing much about buildings and real estate and the property market in Tuscon and how much Mrs Biggs on Escondido Avenue earns. And so everyone was happy. But it turns out that many of those mortgages were sold to people who couldn't really afford them, and that properties were trading hands at values greater than the market could really sustain. Now the ratings agencies should have known this, but they didn't really do a very thorough job of really investigating what was really inside those security bundles - and one could argue that mortages aren't so easily securitizable (that they're not like a commodity like orange juice or steel, where you can empirically test something and know if it's okay or not) but rather that every mortgage and borrower is different in complicated ways, meaning only people who are experts in real estate and who do lots of research should be buying mortgages wholesale. So the collateral that backs the securities is failing, this makes the securities' values plummet. As many institutions have invested a lot (too much) of their own money in these securities, they're taking giant losses. Like all securities, you could get a security broker (an ordinary stockbroker may deal in them), but I don't know if these particular instruments are really sold retail (they're sold in giant chunks from one giant investor to another). 87.114.2.167 (talk) 16:49, 7 September 2008 (UTC)[reply]
Thank you, 87.114.2.167. I learned a lot from your answer. Perhaps you might consider adding a section on "mortgage-backed securities" to Securitization. ៛ Bielle (talk) 16:57, 7 September 2008 (UTC)[reply]

The book Liar's Poker contains an account of the securitization of the mortgage industry for those who don't want to wade through technical papers. DJ Clayworth (talk) 14:55, 8 September 2008 (UTC)[reply]

Rabbinical Judges in Israel Supreme Rabbinical Court

Can any user please tell me where I can find the names of all the Rabbinical Judges who today sit in the Israel Supreme Rabbinical Court in Jerusalem. Thank you. Simonschaim (talk) 19:13, 7 September 2008 (UTC)[reply]

Rabbonim Avraham Sherman, Avraham Sheinfeld, and Chaggai Izrir [18]. - Lambajan 03:55, 8 September 2008 (UTC)[reply]
Basically, all I could find is news articles. I guess the word for judge is dayan, so I googled 'Supreme Rabbinical Court dayanim'. - Lambajan 04:00, 8 September 2008 (UTC)[reply]

Thank you. Simonschaim (talk) 17:49, 8 September 2008 (UTC)[reply]

Timeline of the Sinking of the HMHS Britannic

Is the timeline of the sinking of the HMHS Britannic in this page is the correct one? Aquitania (talk) 20:01, 7 September 2008 (UTC)[reply]

September 8

philosipher who theorized that mandkind was created by "children gods"

Many years ago I read of a philosipher who theorized that mandkind was created by "children gods", who became board and abandoned their creation. I would like to re-visit this philosipher's work, but can not find any information on this theory.

Any help will be appreciated. --Davdum (talk) 02:31, 8 September 2008 (UTC)[reply]

Loans from banks being sold

I was just reading about Fannie Mae and Freddie Mac and it said:

Here's how they work: Banks loan money to home buyers. The banks then sell those mortgages - assuming they meet certain credit standards - to Fannie Mae or Freddie Mac.

Banks then use the money they get from the sale of those mortgages to make new loans. Fannie and Freddie, meanwhile, bundle those loans, attach a payment guarantee to them, and resell them as bonds.


I thought banks usually keep the loans. Dont you get a bill every month from your bank for your mortgage? Doesnt that mean they own the loan? —Preceding unsigned comment added by 75.82.33.31 (talk) 02:42, 8 September 2008 (UTC)[reply]

It depends on the bank. Some banks do hold their own loans. Dismas|(talk) 03:12, 8 September 2008 (UTC)[reply]
You pretty much always make your payments to whoever originally lent you the money, but they will often forward that onto someone else. It's all done in such a way that you never need to know who actually owns your mortgage (it may well have been split between multiple people in weird ways - that's kind of where the trouble started!). --Tango (talk) 03:23, 8 September 2008 (UTC)[reply]
Could you please explain in more detail about how the loans are split inbetween multiple people and how the trouble started. Thanks —Preceding unsigned comment added by 76.90.110.7 (talk) 06:42, 8 September 2008 (UTC)[reply]
One benefit to banks of selling their loans to Freddie or Fannie (or somebody else) is that the bank frees up capital to make new loans. In theory, this bring in profit more quickly than hanging on to the old loan would. There's another effect as well: let's say you have a mortgage balance of $200,000 at 6% interest. Simplifying for the sake of the example (I'm ignoring amortization), your mortgage is an agreement to pay $1,000 a month for the next 30 years (that's the interest; you're also paying a few bucks toward the principle and some money for taxes and insurance).
To an investor, then, your mortgage is just an investment that will return $1,000 a month. If mortgage rates fall to 5%, I might be willing to pay more than the unpaid balance of your mortgage, because it's still going to bring me more money than buying a new, 5% mortgage will. Similarly, if mortgage rates rise to 7%, I'll pay less than the face value your mortage, and the current holder will sell it to me for less, so that the return on my investment gets closer to the 7% I could get from someone else. So there could be three mortgages that started exactly like yours, now held by three different investors; for each of them, the effective return on their own investment is different. You, meanwhile, and the other two people are still paying 6% on your $200,000 mortgage.
As with used cars, the value of your mortgage to an investor is what the investor is willing to pay for it.
As for the monthly bill for your mortgage, in many cases it's handled by a mortgage servicing company; they're just collecting your payment, making the necessary tax payments, and sending the interest to whomever now holds the mortgage. --- OtherDave (talk) 11:04, 8 September 2008 (UTC)[reply]
I don't know the details, but one of the ways the banks divided up mortgages was to group a whole load of mortgages together and then sell two types of contracts secured on those mortgages, a high risk and a low risk one. Each contract entitled you to a share of the returns, with the high risk ones getting a bigger share. Whenever someone defaults on one of the mortgages in the collection, the loss is taken by the high risk contracts, this means the low risk ones have virtually no chance of anyone defaulting. At least, that was the plan. Such large numbers of people ended up defaulting that the high risk contracts were completely used up and the (not so) low risk ones started to lose money. Everyone had assumed the low risk ones were safe and suddenly they weren't, and everything spiralled from there. (I'm sure it's all far more complicated than I make out and there were plenty of other causes, but that's one of them.) --Tango (talk) 11:34, 8 September 2008 (UTC)[reply]

Religious symbology

What religions/beliefs/whatever do these symbols represent? I realize that the image is being used in an article about religion in science fiction and may not be strictly associated with just one religion but get as close to one belief system as you can, please. I originally only wanted to know #11 but I thought I'd expand my mind a bit more and just ask about them all. From 12:00 going clockwise:

  1. Jain
  2. Red Baron ;-) - Lambajan 04:09, 8 September 2008 (UTC) Cross pattée (Thanks 83.250) - Lambajan 04:19, 8 September 2008 (UTC)[reply]
  3. Hands of God - Lambajan 04:12, 8 September 2008 (UTC)[reply]
  4. Triple Crescent - Lambajan 04:26, 8 September 2008 (UTC) -- Taken from the Diane de Poitiers emblem... AnonMoos (talk) 08:31, 8 September 2008 (UTC)[reply]
  5. Sikh - Lambajan 04:05, 8 September 2008 (UTC)[reply]
  6. Buddhism - Lambajan 04:05, 8 September 2008 (UTC) - (Ayyavazhi, see below)[reply]
  7. Jain (again) or Hinduism.
  8. Buddhism - Lambajan 04:05, 8 September 2008 (UTC)[reply]
  9. Baha'i - Lambajan 04:05, 8 September 2008 (UTC)[reply]
  10. Norse (?) -- Sun cross -- AnonMoos (talk) 08:43, 8 September 2008 (UTC)[reply]
  11. Aum Primarily Hinduism - Lambajan 04:05, 8 September 2008 (UTC)[reply]
  12. Islam
  13. Judaism
  14. Shinto
  15. Christianity (general? or is it more specific?) Greek Cross - Lambajan 04:19, 8 September 2008 (UTC)[reply]
  16. Taoism


Thanks, Dismas|(talk) 03:58, 8 September 2008 (UTC)[reply]

I'm pretty sure #11 is a Hindu symbol, and I think #7 is as well. The swatstika is a common symbol of luck in India, at any rate, so I presume it's from Hinduism. --Alinnisawest,Dalek Empress (extermination requests here) 04:01, 8 September 2008 (UTC)[reply]
Number two is the Cross pattée, very similar to the Iron Cross (not the nicest connotations there), which is Christian, I guess (since it's associated with the Crusades) 83.250.202.36 (talk) 04:11, 8 September 2008 (UTC)[reply]
Also, I don't recognise number ten, but it does like faintly runic (although it's not part of the runic alphabet), so it might be Norse. However, much more common symbols for Norse paganism is Thor's hammer Mjölnir and the Valknut 83.250.202.36 (talk) 04:14, 8 September 2008 (UTC)[reply]
Correction: it appears to be a more general symbol for paganism. Also: kinda disappointing that there's no Klingon symbol in there :) 83.250.202.36 (talk) 04:17, 8 September 2008 (UTC)[reply]
And the last one: number four appears to be a wicca symbol. Most of these can be found at commons:Religious symbols, btw 83.250.202.36 (talk) 04:24, 8 September 2008 (UTC)[reply]
Ah.. I didn't even think to check commons for that. - Lambajan 04:29, 8 September 2008 (UTC)[reply]
Also, number six isn't Buddhism, it's Ayyavazhi, whatever that is :) 83.250.202.36 (talk) 04:30, 8 September 2008 (UTC)[reply]
Wow. My misconception is a common one. - Lambajan 04:43, 8 September 2008 (UTC)[reply]
For a quasi-Buddhist lotus symbol, see Flag of Kalmykia... AnonMoos (talk) 09:02, 8 September 2008 (UTC)[reply]

BTW, a "reverse swastika" (卍, U+534D) is a Hanja symbol for "Bhuddist temple." --Kjoonlee 07:50, 8 September 2008 (UTC)[reply]

Another religion question

Is there a name for the idea that the Christian god, the Norse gods, the Egyptian gods, etc are all subordinates under one even more powerful god? Dismas|(talk) 04:00, 8 September 2008 (UTC)[reply]

Pantheism? - Lambajan 04:09, 8 September 2008 (UTC)[reply]
Actually, I was wrong about that. Look at Monolatrism and Henotheism. - Lambajan 04:37, 8 September 2008 (UTC)[reply]
King of the Gods is the concept you're looking for, I think. Although I don't really see how that applies to the Christian god, as he is the only one. 83.250.202.36 (talk) 04:28, 8 September 2008 (UTC)[reply]
Agreed. Most belief systems with many gods don't hold them to be all equal and consider one to be the greatest. The Abrahamic conception holds God to be the only one. I could see how one would reason that you could put them together and say that Zeus refers to God or even Zeus is beneath God, or likewise but replace Zeus with Odin or Horace or whomever, but this only works within the framework of the other religions (or your personal concept of them), but the only way it could work within the framework of how the Abrahamic God is generally understood is by deciding that these other lesser gods were not (or should not be) regarded as gods but something else, perhaps tales personifying attributes of God or maybe prehistorical prophets of God or whatever. Regardless, it's hard to know exactly what conceptions followers of these religions had about the nature of their gods, but any idea you come up with is bound to meet an argument. More likely lots of arguments. Our data is more complete on some cultures over others. It might be an interesting thing to check into. - Lambajan 05:06, 8 September 2008 (UTC)[reply]
I've looked through all three articles suggested thus far and though they're close, none of them seem to state that the other gods, across all religions, answer to a higher god. Note the "across all religions" part. I'm not just referring to the ancient Greeks with Zeus as the head or to the ancient Egyptians with Ra as their head god. But a more supreme god who would rule over both of those plus all the other gods in every religion. And please, 83.250, keep your own religious views out of this. You're not going to convert me to Christianity just by waving your flag here. If anyone has read Job: A Comedy of Justice, that's where I'm mainly getting this question from. Dismas|(talk) 10:05, 8 September 2008 (UTC)[reply]
Oh, trust me, the last thing I'm trying to do is convert you to Christianity, I'm through and through atheist :) (and at a different computer, so I guess my IP will be different...). I'm fascinated by mythology, but I'm not in the least religious (and not in the least missionary, I try to convert no one to no set of beliefs). I misunderstood your question, and thought you were referring to pantheons where there are many gods subject to one chief god (like Odin is the head of the Aesir and Zeus is the head of the Olympians), and I was simply expressing my surprise that you included the Christian god in your list of examples, as the Christian faith preaches that he is the only God, and thus would not fit that mythological pattern.
As for the concept you are looking for, I think the closest thing to what you are referring to is pantheism, as suggested by User:LambaJan, which basically preaches that everything is an aspect of a supreme being of some sort. Look at this beautiful quote (which I'm borrowing from the Hindu section of the article):
This whole universe is Brahman, from Brahman to a clod of earth. Brahman is both the efficient and the material cause of the world. He is the potter by whom the vase is formed; He is the clay from which it is fabricated. Everything proceeds from Him, without waste or diminution of the source, as light radiates from sun. Everything merges into Him again, as bubbles bursting mingle with air-as rivers fall into the ocean. Everything proceeds from and returns to Him, as the web of the spider is emitted from and retracted into itself.
(as I said, I'm through and through atheist, but damn, those Hindus could write!) There are certainly examples of religions that have recognized that other gods exist, but they generally say that the other gods are somehow inferior, and will be defeated by the "True God". If I remember the Old Testament correctly (which it's very possible I might not be, and would be glad to be corrected by someone with more knowledge on the subject), in many of the earlier parts there are implications that other (non-Jewish) deities do exists (like Baal), but that they are nothing compared to the great YHWH, and he and his promised people will drive them away. Another similar (but much more controversial) idea is of course the satanic verses in Islam.
If there is a term for the concept you are referring to, I don't know it. I'd be thrilled to be educated on the subject (as I said, I'm truly fascinated by mythology, in all its forms), and thus leave the subject to my more educated companions :) 195.58.125.53 (talk) 12:14, 8 September 2008 (UTC)[reply]

Short Story where people have a limited number of words per day

I have searched for this on the Internet and on several other "book identification" websites with no luck. I am looking for a short story I read in middle school or maybe elementary school. The writing style, as I remember it, was something like Ray Bradbury or Kurt Vonnegut but I have looked through lists of their stories and if it was by one of them it wasn't immediately obvious to me. The story was about a (futuristic?) society wherein people had only a limited number of words per day that they can say. A man was trying to save up enough words to tell a woman he loves her, but then uses them up or something so when she calls him he can't say anything. That's all I can remember. If someone at the desk doesn't know it, I may try to get it published and wait to get sued because I want to know SO BAD what this story was; I really liked it. 69.244.5.221 (talk) 05:00, 8 September 2008 (UTC)Sariasister69.244.5.221 (talk) 05:00, 8 September 2008 (UTC)[reply]

This reminds me of the poem "The Quiet World" by Jeffrey McDaniel:
In an effort to get people to look
into each other's eyes more,
and also to appease the mutes,
the government has decided
to allot each person exactly one hundred
and sixty-seven words, per day.
When the phone rings, I put it in to my ear
Without saying hello. In the restaurant
I point at chicken noodle soup.
I am adjusting well to the new way.
Late at night, I call my long distance lover,
proudly say I only used fifty-nine today.
I saved the rest for you.
When she doesn't respond,
I know she's used up all her words,
so I slowly whisper I love you
thirty-two and a third times.
After that, we just sit on the line
and listen to each other breathe.
Twas Now ( talkcontribse-mail ) 06:37, 8 September 2008 (UTC)[reply]

Oh my gosh I think that's it! I guess I had a few of the details wrong, but I'm so glad I finally found it!69.244.5.221 (talk) 12:59, 8 September 2008 (UTC)Sariasister[reply]

artist of this work?

Who is the artist for the image shown in Declaration of the Rights of Man and of the Citizen? I do not doubt its PD status, given its outdated use of the word "Francois," but I need it for an image bibliography lol. 199.111.202.129 (talk) 07:59, 8 September 2008 (UTC)[reply]

From here (http://www.bridgemanartondemand.com/art/102120/Declaration_of_the_Rights_of_Man_and_Citizen_1789) it just seems to put the artist as 'French School' not sure if that's enough 194.221.133.226 (talk) 11:56, 8 September 2008 (UTC)[reply]

FTSE 100 opening and closing figures

The BBC business page is currently showing the FTSE 100 as having increased by 3.81%

However, the Yahoo! finance page shows only a 1.46% increase.

I see that Yahoo! are basing the percentage increase on last Friday's close, whereas the BBC are basing it on this morning's opening figure. To me the Yahoo! representation is the more logical, but I'm utterly puzzled as to how a morning's opening figure could differ from the previous trading day's closing figure.

Can anyone enlighten me?--85.158.139.99 (talk) 08:48, 8 September 2008 (UTC)[reply]

Here's an article on it... http://www.usatoday.com/money/perfi/columnist/krantz/2008-02-20-stocks-open-lower_N.htm . 194.221.133.226 (talk) 10:14, 8 September 2008 (UTC)[reply]
That USA Today article indicates why a given stock/share can trade at a quite different price when the market opens compared with the previous close, but it doesn't tell me why the index itself would have a different value at opening than it had at the previous close.--85.158.139.99 (talk) 10:36, 8 September 2008 (UTC)[reply]
If Company X is trading at 100 (people are willing to buy and sell at that amount) when the market closes it's closing price will be 100. If something happens to the company (like price of the raw goods it purchases increase) while the market is closed it may seem less profitable to investors, so when the markets re-open sellers will have put a lower price to entice people to buy. So it may open at, say 60. The FTSE 100 is just the combination of a hundred such companies, whose fortunes may have changed overnight while the stock market was closed. - Phydaux (talk) 12:36, 8 September 2008 (UTC)[reply]
The index is the price of all the individual companies stock-prices within that index combined. So the FTSE 100 is the combined share-prices of all the top 100 companies. It's not a piece of stock itself. 194.221.133.226 (talk) 10:50, 8 September 2008 (UTC)[reply]
Sorry should add you might want to have a look at stock index 194.221.133.226 (talk) 10:52, 8 September 2008 (UTC)[reply]
Thanks for your responses, though I have to confess I'm still puzzled. Although I was aware that share indices like the FTSE 100 are simply some sort of average of all its constituents, what puzzles me is the calculation. I see from the Stock market index article that generally constituents are weighted according to their capitalisation (which seems rational enough to me).
What I don't see is how an opening index can have a value different from its previous closing value. If the opening value is calculated from the first few trades, how on earth do they do it? Do they look at the first share traded, compare the share's percentage change from its previously traded value, and apply that to the whole index? It can't be that, as it would clearly lead to large biases. Or do they wait for a few minutes to get a more representative set of transactions? Well, possibly that's it.--85.158.139.99 (talk) 12:48, 8 September 2008 (UTC)[reply]
All FTSE 100 shares are very heavily traded, it won't take long for them all to have had at least one trade, as far as I know the opening index waits until they have, which is likely just a few seconds to a minute. --Tango (talk) 13:16, 8 September 2008 (UTC)[reply]
Tango, thanks for the sensible answer which shows that somebody here understands my question.--85.158.139.99 (talk) 13:50, 8 September 2008 (UTC)[reply]
Based on the understanding that the 'open price' is the price of the first trade of a stock at the start of the day (and the close price is the price as at 4.35pm for the FTSE 100), then all it takes is a number of stocks to have a different 'open' price to the 'close' price for their to be discrepency between the examples you chose. As the article suggests - if a stock closes at £1.00 a share and it transpires during the markets close that the company has dropped profits by 50% then anybody purchasing the stock the next day could expect to do so at a reduced price. Given that knowledge the first trade may occur at say 90p (10% down). If 10 stocks each open at a different price (up or down) then that will have some effect on the 'open' value of the FTSE 100 index. The index's value changes with the stock and its 'open' value will presumably be based on the open-price of the first trade of each of the FTSE 100 constituents, whilst its close price will be similarly based on the closing-price of each of the FTSE 100 firms. The difference will arise because new information/knowledge becomes available even while the market is closed - therefore the first trade of a day for some stocks will be different to the close-price. 194.221.133.226 (talk) 13:13, 8 September 2008 (UTC)[reply]
I am fairly certain that the FTSE-100 index is calculated from quoted prices of its constituent stocks (i.e. average of closest bid and offer quotes from different sources) rather than on traded prices. Yesterday afternoon I liked your car and offered you $1000 for it. This morning I like it a bit less and offer you only $900. It's that simple. Gandalf61 (talk) 13:34, 8 September 2008 (UTC)[reply]
I think the quoted price is usually the price the stock last traded at, which will generally be someone at or between and bid and ask prices. On stocks as heavily traded as a FTSE 100 stock, the bid-ask spread is likely to be very small, so whatever way you measure it the difference will be a fraction of a penny. --Tango (talk) 13:47, 8 September 2008 (UTC)[reply]
Yes, the stock price is not what traders have to buy and sell at, but rather a reflection of what traders have been buying and selling them for. That is why the last sale (close) could be high, and the next day the first sale could be low. The sale isn't affected by any previous price, or the price you see on the BBC ticker. - Phydaux (talk) 13:51, 8 September 2008 (UTC)[reply]
The London Stock Exchange main market is a quote-driven market, not an order-driven market. This means that market makers make two-way (bid and offer) quotes on a range of stocks on their books, up to a given size of trade. So, yes there is a clear difference between quoted prices and traded prices. Investors are usually more interested in the quoted price of a stock because that is the price available in the market now, as opposed to the price at which someone last traded. It is entirely possible for an illiquid stock to have a published quoted price even if it has not been traded today (although, as has been said, this wouldn't happen in practice for a FTSE-100 stock). Gandalf61 (talk) 14:09, 8 September 2008 (UTC)[reply]
Ah, now I see the confusion, the word "quote" has two meanings in this contest. You're talking about the prices quoted by money-makers, I'm talking about the prices quoted by Yahoo! Finance, and the like. I think we're both right! --Tango (talk) 14:29, 8 September 2008 (UTC)[reply]
Additional question. Let's see we have an equally-weighted index of two stocks (Hmm, X and Y) and X closes at $2 while y closes at $1 with the index at 1.5 at the close. The next morning, before the market opens, company X announces that they've been misstating their earnings for ten years so that the CEO can get big bonuses (this is a real life example!). Nothing new happens for Y which is in an unrelated industry. Y opens at 9:30 with a price of $1 and the price does not change till 10:01 (but the stock trades). Trading in X is delayed and does not start till 10am when it opens at $0.50. So, at 10am, the value of the index is 0.75. Q: What is the value of the index between 9:30 and 10? --Regents Park (count the magpies) 13:48, 8 September 2008 (UTC)[reply]
The closing value of the day before. - Phydaux (talk) 13:53, 8 September 2008 (UTC)[reply]
(To expand on my comment) The index is just a reflection of the trades that have taken place, as no trades can take place while the stock market is closed the value doesn't change. But I would imagine many speculators, investors and brokers would assume a fall in the value of the company, and the index, and trading would open lower than it closed the day before. - Phydaux (talk) 13:59, 8 September 2008 (UTC)e[reply]
That's not strictly correct. The calculated and reported open price will be 0.75 since that is the average of the two open prices. However, this is what will be reported after the fact when the exchange reports (Open, High, Low, Close) for the day. I'm curious about how the index will be computed between 9:30 (when Y and the market open) and 10 (when X opens). My guess is that the close of X will be used and the index will show a drop at 10. (Index futures will, of course, reflect the true price of X.)

Masochistic behavior...What is this?

I have all the info. about sadistic behavior but what about masochistic? Not all masochists enjoy humiliation. I enjoy doing all for my spouse who has DSM. but i hate it when he calls me names or humiliates me. Its not just masochism. Its like I want to be his slave.I enjoy cooking, cleaning, anything he wants from me I do it because i love it. I like getting into physical fights with him. The pain hurts but its not mistaken for pleasure. It DOES hurt. I can inflict pain on myself but i cant draw blood because im afraid it will hurt. When im away from my spouse I feel that I should stop all action because Hes not with me and it doesnt feel right.when i cook for others or clean for others i feel out of place. with all of this information, What is this disease? I mean it has to be a disease if DSM could be counted as one. --Chaela <3 (talk) 19:14, 8 September 2008 (UTC) —Preceding unsigned comment added by Chaela89 (talkcontribs) 19:06, 8 September 2008 (UTC)[reply]

I'm sorry that the Reference Desk cannot diagnose psychiatric or other medical problems. I strongly recommend that you consult with a mental health professional or perhaps a social worker. Marco polo (talk) 19:55, 8 September 2008 (UTC)[reply]
I thought he was just your fiance? — Lomn 00:13, 9 September 2008 (UTC)[reply]

North Korea

Today, it's easy to observe that South Korea and it's capitalistic/democratic system is economically superior to North Korea's communist system. With this in mind, why doesn't North Korea change to a governing system more similar to South Korea's? ScienceApe (talk) 20:13, 8 September 2008 (UTC)[reply]

Because I highly doubt this guy would agree to it! --Alinnisawest,Dalek Empress (extermination requests here) 20:35, 8 September 2008 (UTC)[reply]
Yea I agree. But does he have so much power that no one in Korea can oppose him? ScienceApe (talk) 21:39, 8 September 2008 (UTC)[reply]
Anyone there with power has their power because he lets them have it. No one (or very few) who have power want to risk losing that power. — Twas Now ( talkcontribse-mail ) 21:44, 8 September 2008 (UTC)[reply]
Is Kim Jong the only thing that is preventing change? ScienceApe (talk) 22:21, 8 September 2008 (UTC)[reply]
Well, the people there pretty much consider him a deity. It's not because they're stupid or anything, but they've been brainwashed into thinking that since they were babies. So the attitude of the people would also hamper change. If an assassin were to take out Kim Jong-il, I doubt North Korea would immediately change over to a democratic government. If South Korea was to invade North Korea and forcibly install a democratic government, that might work. I'm certainly not advocating either point of view, though!
Also, Kim Jong-il very likely has his successors all planned out who will continue the "Communist" government (it's not really communist, more fascist), so his death would likely not change much. --Alinnisawest,Dalek Empress (extermination requests here) 22:26, 8 September 2008 (UTC)[reply]
According to this, we might soon get to see if you're correct. GreatManTheory (talk) 17:42, 9 September 2008 (UTC)[reply]
I think many or most Korea experts and Koreans believe that the two states would have been unified and neutralized years ago if not for American opposition.John Z (talk) 22:45, 8 September 2008 (UTC)[reply]
Yes, they would have. --Carnildo (talk) 00:13, 9 September 2008 (UTC)[reply]
It's worth remembering that systems like this rarely justify themselves on being truly economically superior, or at least never acknowledge that it has been a "fair fight" (and often it hasn't been—if you tried to run a capitalistic system with the sorts of sanctions that are put against North Korea, you'd have trouble too). Another way to think of this is: the economies of autocratic states are often much better than those of democracies (for various reasons, often relating to large reserves of natural resources in said autocratic states). Would that fact at any point in time convince those in said democracies that they should convert to the autocratic system? Or would they say, "oh, that's just fluctuations of the market" or "oh, that's because they're setting unfair oil prices" or "oh, that's because of X and Y and Z"? I doubt it. --98.217.8.46 (talk) 23:24, 8 September 2008 (UTC)[reply]
List_of_countries_by_GDP_(nominal) certainly seems to rebut your assertion that "economies of autocratic states are often much better than those of democracies." Unless, of course, your conception of "good" or "bad" economies is based on something other than economic growth. GreatManTheory (talk) 02:15, 9 September 2008 (UTC)[reply]

Instead of North and South Korea, imagine the question is the Mafia and Disney. Why won't the Mafia simply take the honest road? Well, maybe they don't know how; they don't want to risk losing what they have; they don't want to get arrested for what they did in the past; or they really believe in what they do. DOR (HK) (talk) 01:45, 9 September 2008 (UTC)[reply]

I don't think it's right to say that North Korea's system persists only or even mainly because everyone with any privilege owes their position to Kim Jong-il. I'm not sure how true that is in practice. I think that the (very powerful and privileged) military may have its own system for promotions in which Kim is not involved. I think that the system survives because it gives tremendous power and privilege within North Korea to that country's military, and especially the military elites and security services, who very effectively suppress any hint of dissent and who could easily crush any nascent opposition force. Marco polo (talk) 16:07, 9 September 2008 (UTC)[reply]

Secular Humanism -Dead or Alive? (Insert spinning joke here)...

Hello Wikipedia,

As an non-american, this election is really gripping. Obviously its easy to despair at points like these (when you're about to elect a few more far-right loons for office) but i'm trying to stay positive. Michael Moore tells me that the majority of americans hold liberal views points (on things like abortion, gays, healthcare etc..) whereas that guy who used to head up 'Focus on the Family' (<blp violation removed>?) tells us that 'Secular Humanism is dead' at that when the religious right vote, they decide the election. So which is it? Are there some surveys or something anyone can direct me to or something with a little more NPOV? Thanks 82.22.4.63 (talk) 20:28, 8 September 2008 (UTC)[reply]

I have removed a BLP violation in the above comments. James Dobson, head of Focus on the Family, does recommend that fathers take showers with their sons to make sure that they don't become gay. Corvus cornixtalk 20:48, 8 September 2008 (UTC)[reply]
Whoa, Corvus. You removed a 'BLP violation' and then you wrote that about James Dobson without a reference? maybe you should rethink. Do you have a reference for that remark? DJ Clayworth (talk) 14:59, 9 September 2008 (UTC)[reply]
[19]. Corvus cornixtalk 18:49, 9 September 2008 (UTC)[reply]
Whoa. Stop. Rewind. Fathers taking showers with their sons keeps the sons from becoming gay? Exactly how is that supposed to work? -- Captain Disdain (talk) 07:41, 9 September 2008 (UTC)[reply]
Yes, I'd be more worried about the fathers who wanted to do that, than about the sons. Well, I personally wouldn't be worried about the sons at all - that is, until the father climbed into the shower with them. Hoo boy. We've got a way to go yet. -- JackofOz (talk) 14:00, 9 September 2008 (UTC)[reply]
Note that Corvus hasn't supported his allegation with anything yet. We only have his word for it. DJ Clayworth (talk) 14:59, 9 September 2008 (UTC)[reply]
These two are not mutually exclusive. The majority of Americans can be generally liberal, and the religious right can decide the vote. This just suggests that the majority of Americans are also apathetic toward politics. — Twas Now ( talkcontribse-mail ) 21:48, 8 September 2008 (UTC)[reply]

Two key issues: which side motivates their supporters to get out and vote; and how honest the vote count is. DOR (HK) (talk) 01:47, 9 September 2008 (UTC)[reply]

There have been a number of reports recently, including pieces by NPR and the BBC, that propose that evangelicalism is a fading force in 2008 America. I’ll try to those news stories for you, but it was a while ago so I’m not sure that I’ll secede. Here’s and interesting book on the mater however. These last years have suitably been a terrifying ride, but the feeling today is that The Dark Side is, if not beaten, in retreat. In a few decades this time may be looked at in a similar way to how we view the Civil Rights Era, a horrible part of our past that is remembered and commemorated as an example of how the American people can overcome evil. There are strong paroles (the current open acceptance of homophobia in the media for instance). --S.dedalus (talk) 03:05, 9 September 2008 (UTC)[reply]
Oh, and secular humanism is defiantly not dead. An Imagine No Religion billboard just went up in downtown Seattle and there are dozens more around the country. :) --S.dedalus (talk) 03:07, 9 September 2008 (UTC)[reply]

Thanks for those reponses guys -its quite a relief -although i'd be interested in what the response would be over on good ol' Conservapedia. (P.s. SDedalus - i love the 'straight not narrow' thing on your page -genius!) anyway, two questions, 1) What does BLP stand for and 2) it was actually a serious question -can't you just say something like 'that has yet to be proven' rather than delete it from record?82.22.4.63 (talk) 18:46, 9 September 2008 (UTC)[reply]

"BLP" is Wikipedia-talk for a violation of the Wikipedia policy on biographies of living people. Corvus cornixtalk 18:51, 9 September 2008 (UTC)[reply]

Qur'an and the term "we"

I was recently watching a TV show about how the Qur'an "predicted" current scientific facts 1400 years ago. In this program, there were quotes of the Qur'an to suppoot the thesis that the Qur'an made such predictions (and that this is "proof" in the existance of God. In many of the verses used, there was reference to "we." My understand that Islam is monothestic but "we" is plural. Since Islam does not promote a triune godhead (as does Christianity), who is the "we" that is being referred to? What is the context of this plural? —Preceding unsigned comment added by 69.77.185.91 (talk) 21:21, 8 September 2008 (UTC)[reply]

It could be the royal we. Remember, you were hearing translations from the Arabic, there is often a fair amount of choice in how you translate things, the translator may have decided that "we" more accurately conveys the feel of the verse even if a literal translation would have had "I". --Tango (talk) 22:45, 8 September 2008 (UTC)[reply]
In Genesis, the Hebrew god is sometimes referred to as Elohim, which is plural in form, but considered singular in meaning. At at least one point, the deity says "Let us... " which is interpreted in many ways, as the royal we, as a god in a polytheistic system, or as reference to the Elohim title, among others. —Preceding unsigned comment added by Steewi (talkcontribs) 03:39, 9 September 2008 (UTC)[reply]
I just glanced through a few sections of my Qur'an and skimmed to where I found a few instances of this 'We'. A couple times they were Muhammad speaking and a couple times they were quotations of Moses. In either case it was there in the Arabic as well. Muslims have a clear distinction between the Prophets and God. They would never say a Prophet is God or is on the same level, but nevertheless this type of speech is there. It's maybe taken as sort of a 'royal we', but maybe taken as sort of a 'myself and God' sort of a thing, because even though they're taken as being different and unequal, they are taken as being in concord. If the Prophet is God's vessel and God is speaking through Him then such a conjugation is not illogical. - Lambajan 04:29, 9 September 2008 (UTC)[reply]

September 9

DSM and masochism.

okay if we are being specific. Then if DSM is when a man or woman is dominant and just does strange things to people for their personal gratification then what is it when a person loves to be a slave but hates gratification. Its like they enjoy being a masochist its just that certain things in the symptoms of this masochistic disease just arent things that they want or feel. Is that still masochism? Im not asking for a diagnosis because im pretty sure this disease is what it is. I just want to know if it has a name. Wanting to be a physical slave and do phyisical labor for their "master" AND wanting pain enflicted on them in a sexual reference. What is this called?

--70.42.211.4 (talk) 11:58, 9 September 2008 (UTC)[reply]

FWIW, one of our articles states:

The BDSM term is a portmanteau acronym intended to take in all of the following activities:

You appear to be describing masochism. I'm not sure / I doubt that masochism per se is a disease. DSM is not "when a man or woman is dominant", it indicates three things, dominance & sadism, and masochism. --Tagishsimon (talk) 12:04, 9 September 2008 (UTC)[reply]

What you describe also has a good "submission" component. — Twas Now ( talkcontribse-mail ) 12:07, 9 September 2008 (UTC)[reply]

yes but what if they dont want the "master" to lift a finger, but somtimes they do. Its just a mood that over takes them now and then that makes them just want to be submissive in everyway possible but other times be pure dominant. can you really have such strong lifestyles in both? i know its sadomasochism but isnt that wanting both at one time. what is it when you dont relize that you feel that way? If its not a disease and you dont know you feel that way then what can it be? its not a sexual fantasy because you feel it all day long. —Preceding unsigned comment added by Chaela89 (talkcontribs) 12:12, 9 September 2008 (UTC)[reply]

You can choose to be a sadist. You can choose to be a masochist. You can choose to eat an apple. You can choose to watch a film. You can watch films all day. None of these are, as far as I know, a disease. They are a choice presumably based on factors such as the cost of the activity and the gratification it gives you. You appear to vacillate between thinking that those involved in S&M are sometimes S an sometimes M, or are M all the time. I'm sure there are examples of both types (and of the permanent S type, as well). "what is it when you dont relize that you feel that way?" Lack of self awareness? --Tagishsimon (talk) 12:39, 9 September 2008 (UTC)[reply]
I really think you should read a couple of good books about BDSM. Obviously you've got certain needs, and obviously you don't know how to deal with them -- yet. That's most likely not a medical condition; having urges like that is typically not indicative of a disease. It's just a kink. Kinks are fairly commonplace and not such a big deal; you seem distressed about this, which is understandable, since you're finding out pretty fundamental things about yourself, but you really shouldn't worry. Don't freak out! Kinks aren't a bad thing -- if anything, they probably make you more interesting. But as you're obviously not an experienced kinkster, you should strive to get educated about this stuff... so, read up on it.
Yes, a lot of people can and do switch between a dominant and submissive role. No, even if you're into that kind of stuff, that doesn't mean that you're necessarily into it all the time, or into all aspects of it. There are people who just want to be spanked, for example, or to be verbally humiliated, or immobilized, or simply to be told what to do, whereas some people really want the whole shebang in all its forms. These are personal preferences that you don't need to justify any more than you need to justify whether you prefer coffee or tea.
Our article on BDSM is pretty informative, and so are the articles it links to. Safe, sane and consensual is probably something you want to take in. Just take it easy, and take the time to learn; you'll probably find out a whole lot about what you want and about what you don't want, and both of those are very valuable things to you. Also, the columns of people like Dan Savage and Mistress Matisse are likely to provide a lot of useful perspective. Just relax, you'll figure this out. There's no reason to think that you're not going to be okay, and there isn't anything wrong about having urges like this. If that's how you happen to be wired, hey, more power to you. -- Captain Disdain (talk) 13:01, 9 September 2008 (UTC)[reply]

Reconstructed literary and musical works

I've been having a discussion over at Talk:Felix Mendelssohn#London cab about a report I read that he left the only available copy of his incidental music to "A Midsummer Night's Dream" in a London cab and had to rewrite the entire score from memory. As far as we can tell so far, there's no basis to this story. However, it got me thinking. We've mentioned 2 real cases where writers did have to start from scratch all over again, but I'm sure there are numerous other cases. The same might apply to composers whose scores were lost or destroyed before they were ever printed, and they had to be recreated from memory. Or painters. Any ideas, folks?

If we had an article with a list of such details, what would be a better title than the one I jocularly suggested? -- JackofOz (talk) 14:00, 9 September 2008 (UTC)[reply]

List of works rewritten from memory --Tagishsimon (talk) 14:45, 9 September 2008 (UTC)[reply]

The Catholic Church and death before Jesus

Does the Catholic Church have (or has it ever had) a stance on what happened to people who died before Jesus was born? Did Jews go to heaven and no-one else, or did everyone go to hell? Have they ever taken a position with regards to people who didn't have the opportunity to learn about Christ? Thanks in advance. 90.192.223.228 (talk) 14:29, 9 September 2008 (UTC)[reply]

The resurrection is trans-temporal. A simple answer is that, at Jesus' resurrection, everyone was saved (those who had died prior to the resurrection and those who would be born after). A more complicated answer is that God exists outside of time, so the question has no meaning as it implies false restrictions (i.e., linear time) on a greater reality. Wikiant (talk) 14:35, 9 September 2008 (UTC)[reply]
You may find Limbo informative. DuncanHill (talk) 14:38, 9 September 2008 (UTC)[reply]
This question made me wonder, assuming we evolve, at what point did we gain souls? But I'm thinking that that's a bit of a juvenile a-ha sort of a question arising out of the problem of squaring evolution and faith in the supernatural tales, and or a misunderstanding about what christian religions think a soul is. So scrap that question. --Tagishsimon (talk) 14:44, 9 September 2008 (UTC)[reply]
There's an exciting scene in Dante's Inferno where Jesus mounts an invasion of Hell to rescue Moses and guys like that who had been waiting around for a long time. It was like a Missing in Action (film) prequel. See Harrowing of Hell. --Sean 14:54, 9 September 2008 (UTC)[reply]
The next-to-last paragraph of Salvation#Roman Catholicism seems to answer your question. -- BenRG (talk) 15:13, 9 September 2008 (UTC)[reply]

I know that there are different types of religion with the same name like paganism with A higer power, Or believing in the earth and so on and so forth. What category would this religion fall under: Beleiving in God and Satan not as enemies but as partners. When the earth was created it was created with Good and Bad. (Rocks can be beauriful but you can also throw one at someones head.)

beleiving that we were put here to see how far we could get from evolution. Like a bet. That is how people have happiness and anger. whichever the person holds more of God or Satan has more power over their head and heart. and satan is not neccesarily evil. Just grumpy and in charge of peoples feelings of anger and sadness.